SlideShare uma empresa Scribd logo
1 de 82
ENDOCRINOLOGIA
ENARM
PATOLOGIA HORMONAL
• Cual es el cuadro clínico del hipopitituarismo?
1) Astenia
2) INTOLERANCIA AL FRIO
3) Hipoglucemia
4) AMENORREA
5) Disminución de la libido
6) Ausencia de bello axilar
• Cual es la causa mas común de sobreproducción de hormonas de la hipófisis anterior?
• R = Un adenoma hipofisario
• Como determinas de acuerdo a la medida de un adenoma hipofisario si es micro o macroadenoma?
• R = Mas de 1 cm es macro adenoma, lo contrario es micro
• De los adenomas hipofisarios cual es el tipo histológico mas común?
• R = 1 ero PRODUCTOR DE PROLACTINA 30 % , 2do Productor de GH
• Que manifestaciones clínicas se asocian a la hiperprolactinemia?
• R = AMENORREA, GALACTORREA, perdida de la libido, ABORTOS DE REPETICIÓN, ESTERILIDAD, datos de
hiperandrogenismo como HIRSUTISMO Y ACNÉ.
• Que tipo de fármacos o padecimientos producen hiperprolactnemia?
• R = PROLACTINOMA. Antagonistas de los receptores de dopamina como los NEUROLÉPTICOS (fenotiazinas,
HALOPERIDOL) y ANTIGUOS ANTIHIPERTENSIVOS COMO LA RESEPRINA que inhiben el almacenamiento del
mediador. También los ESTRÓGENOS, la IRA y el HIPOTIROIDISMO.
PATOLOGIA HORMONAL
• A que se le conoce como síndrome de Nelson?
• R = Es el resultado de la perdida del EFECTO INHIBIDOR DE LOS CORTICOSTEROIDES SUPRARRENALES sobre un microadenoma
corticotrofo preexistente.
• Que ocasiona una apoplejía hipofisiaria y como se manifiesta?
• R = Una HEMORRAGIA brusca en la glándula que ocurre a menudo secundario a adenoma. Se presenta de manera brusca con
un INTENSO DOLOR DE CABEZA, DIPLOPÍA E HIPOPITUITARISMO.
• Que defecto genético en niños se relaciona a la deficiencia de hormonas hipofisarias?
• R = Deficiencia en el gen que codifica ¨PIT-1¨
• Cual es la forma de presentación más común hiposecreción de la pituitaria anterior?
• R = AMENORREA
• Cuales son los receptores hormonales mas importantes a buscar para iniciar el tratamiento de cáncer de mama?
• R = Estrogenos y progesterona.
• Que puede ocasionar en hombres elevación de la prolactina?
• R = Ausencia de erección con disminución del grosor del pene.
• Que manifestaciones clínicas da el déficit congénito de leptina?
• R = Obesidad, esta se encarga de dar la SENSACIÓN DE LA SACIEDAD.
• Como actúa la hormona de alfa melanocito?
• R = Es un neuropeptido que REGULA LA INGESTIÓN DE ALIMENTOS
HIPERPROLACTINEMIA
• Cual es una hormona inhibidora de la prolactina?
• R = DOPAMINA, el aumento de estrógenos la inhibe.
• Cuales son los signos y síntomas de hiperprolactinemia?
1) Hombres: DISFUNCIÓN ERÉCTIL, HIPOGONADISMO HIPOGONADOTROFICO, INFERTILIDAD y
disminución de la libido. En ocasiones GINECOMASTIA SIN GALACTORREA.
2) Mujeres: OLIGOMENORREA, amenorrea, GALACTORREA, si el hipogonadismo no es tratado da osteoporosis.
• Que lab se solicitan en hiperprolactinemia y que padecimientos se encuentran relacionados?
• R = HCG en caso de embarazo, HIPOTIROIDISMO, IR y cirrosis.
• Que estudio realizas en hiperprolactinemia no asociada a hipotiroidismo, embarazo o inducida por fármacos?
• R = IRMbuscando micro <2 cm o macroadenoma hipofisiario >3 cm.
• Que medicamentos no deben recibir las mujeres con prolactinomas?
• R = ESTRÓGENOS (<2cm no afectan), TETOSTERONAo EMBARAZO.
• Cual es el tratamiento para hiperprolactinemia?
• R = BROMOCRIPTINA. Antagonistas de la dopamina como CENTENO O CABERGOLINA al acostarse,
Quinagolida a los intolerantes derivados del cornezuelo de centeno
SINDROME METABOLICO
• PARA EL DIAGNOSTICO DE ESTE SÍNDROME ES
NECESARIO LA PRESENCIA DE 3 O MAS DE LOS
SIGUIENTES CRITERIOS:
– OBESIDAD ABDOMINAL MUJERES > 88 CMS Y HOMBRE >
102 CMS.
– TRIACILGLICERIDOS >O IGUAL A 150 MG/DL O CON
TRATAMIENTO.
– HDL <40 MG/DL (HOMBRES) Y < 50 MG/DL (MUJERES).
– TENSIÓN ARTERIAL SISTÓLICA >O IGUAL A 130 MMHG Y
DIASTÓLICA >O IGUAL A 85 MMHG.
– GLICEMIA > 110 MG/DL O CON TRATAMIENTO
HIPOGLICEMIANTE.
SSIADH
• En que región del hipotálamo se secreta la ADH y que funciones tiene?
1) En el NÚCLEO SUPRA ÓPTICO
2) La hormona antidiurética (ADH), o arginina vasopresina (AVP), es una hormona LIBERADA
PRINCIPALMENTE EN RESPUESTA A CAMBIOS EN LA OSMOLARIDAD SÉRICA O EN EL
VOLUMEN SANGUÍNEO. También conocida como argipresina. Hace que los riñones conserven agua
mediante la concentración de orina y la reducción de su volumen, estimulando la reabsorción de agua.
Recibe su nombre de esta importante función como regulador homeostásico de fluidos.
• Cual es la forma mas común de HIPONATREMIA NORMOVOLEMICA?
• R = SSIADH
• Cual es el cc del SSIADH?
1) Neuromusculares: IRRITABILIDAD, debilidad muscular, letargo, CONFUSIÓN y coma.
2) Gastrointestinales: NAUSEA, VOMITO y anorexia
3) La intensidad de los síntomas se relaciona con el grado de hiponatremia
• Como diagnosticas SSIADH?
1) Por exclusión
2) Prueba de sobrecarga hídrica y hallazgos de niveles plasmáticos inapropiadamente elevados de vasopresina
en relación a la osmolaridad plasmática
• Cual es el manejo del SSIADH?
1) RESTRICCIÓN HÍDRICA 500-1000 ml/24 hrs
2) DEMECLOCICLINA (inhibe la respuesta renal a la vasopresina)
PATOLOGIA DE TIROIDES
• En que pacientes aumenta el riesgo de que un nódulo tiroideo palpable sea
maligno?
1) Antecedentes de radiación de cabeza y cuello,
2) AHF de cáncer tiroideo o asociada a linfadenopatia.
• Que patologías te pueden causar hipertiroidismo?
• R = Graves, bocio multinodular toxico, nódulos hiperfuncionantes, tiroiditis sub
aguda.
• Cuales son los signos y síntomas del bocio endémico?
• R = Focos subesternales pueden ocasionar: Compresión traqueal, insuficiencia
respiratoria, sx de vena cava superior, hemorragia GI + varices esofágicas, parálisis
de los nervios frénico, faríngeo recurrente o sx de Horner.
• Cuales son los datos de lab de bocio endémico?
• R = T4 sérica suele ser normal, TSH normal, Sin embargo varían con
hipo/hipertiroidismo
PATOLOGIA DE TIROIDES
• Que patologias o farmacos pueden ocasionar bocio en hipotiroidismo?
• R = Tiroiditis de Hashimoto, deficiencia de yodo, FÁRMACOS BOCIOGENOS (LITIO, yoduro, propiltiuracilo,
metimazol, fenilbutazona, sulfonamidas, AMIODARONA) , Tiroiditis sub aguda de Quervain
• En que situaciones el bocio se encuentra ausente en hipotiroidismo?
• R = Secreción hipofisiaria disminuida de TSH, Destrucción de la glándula por cirugía o radiación.
• Cual es el cuadro clínico del hipotiroidismo?
1) Tempranos: fatiga, LETARGO, debilidad, artralgias, mialgias, calambres musculares, INTOLERANCIA AL
FRIO, ROTS <.
2) Tardios: dislalia, ESTREÑIMIENTO, edema periférico, palidez, RONQUERA, AUMENTO DE PESO,
amenorrea-menorragia, GLOSITIS.
3) DERRAME PLEURAL, PERITONEAL O PERICARDICO. CORAZON DE MIXEDEMA POR DERRAME
PERICARDICO.
• Cuales son las complicaciones del hipotiroidismo?
1) Cardiacas
2) Locura del mixedema: psicosis orgánica con delirios paranoides, coma mixedematoso desencadenado
por infeccioso o traumatismo que se acompaña de hipo/glucemia-natremia.
• Cual es el manejo del hipotiroidismo?
• R = Levotiroxina
PATOLOGIA DE TIROIDES
• Que características clínicas acompañan a la tirotoxicosis?
• R = Nerviosismo, PALPITACIONES, TAQUICARDIA, fatiga, debilidad, pérdida de peso con buen
apetito, DIARREA, INTOLERANCIA AL CALOR, PIEL CALIENTE, transpiración excesiva, labilidad
emocional, alteraciones menstruales, TEMBLOR FINO EN LAS MANOS, alteraciones oculares
y aumento variable de la glándula. FA.
• Como se manifiesta la CRISIS O TORMENTA TIROIDEA?
• SE MANIFIESTA POR DELIRIO, TAQUICARDIA, VOMITO, DIARREA y FIEBRE.
• Cual es el manejo de la tormenta tiroidea?
• R = Dosis altas de PROPILTIUORACILO, yodo o contrastes yodados, PROPANOLOL
Y DEXAMETASONA
• Cuales son los análisis mas específicos para enfermedad de Graves?
• R = TSH-R AB 2da generación y TSH-R recombinante son > 75%
• Que diferencia hay entre la captación de yodo de las siguientes patologías?
1) Enfermedad de Graves y bocio multinodular toxico: ALTO
2) Tiroiditis sub aguda: BAJO
PATOLOGIA DE TIROIDES
• La enfermedad de GRAVES es la causa mas común de TIROTOXICOSIS, que la caracteriza?
1) Enfermedad de Basedow en Europa.
2) ETIOLOGÍA AUTOINMUNE, aumento de hormonas tiroideas, AUMENTO DE GLÁNDULA, AC VS TSH.
3) Mujeres 8:1. SE RELACIONA con enfermedades autoinmunes como ANEMIA PERNICIOSA, MIASTENIA GRAVE.
4) Asociación con HLA-B8 y HLA DR3, AC VS PEROXIDASA, AC VS TIROGLOBULINA y ANA están presentes
• Que es la enfermedad de Plummer en tirotoxicosis?
• R = Son adenomas tóxicos tiroideos simples
• Que es la enfermedad de Jodbasedow en tirotoxicosis?
• R = Hipertiroidismo inducido por yodo
• Que patologías extraglandulares originan tirotoxicosis?
• R = Aumento en la HCG, EMBARAZO MOLAR, CORIOCARCINOMAy neoplasias testiculares.
Administración de AMIODARONA en arritmias.
• Cuales son los signos y síntomas de tirotoxicosis?
1) Mirada fija, CAÍDA DE PARPADO, taquicardia o FA, TEMBLORES FINOS, PIEL HUMEDA/CALIENTE.
2) GRAVES: Oftalmopatia con quemosis, conjuntivitis y PROPTOSIS. DERMOPATIA “MIXEDEMA PRETIBIAL”
con ACUMULACIÓN DE GLUCOSAMINOGLUCANOS que ocasiona edema con textura rugosa.
PATOLOGIA DE TIROIDES
• Cual es el manejo de la enfermedad de Graves?
• 1. PROPANOLOL: taquicardia, temblores y ansiedad
• 2. Derivados de la tiourea (METIMAZOL O PROPILTIURACILO): Puede causar
AGRANULOSITOSIS
• 3. Metimazol: < riesgo de necrosis hepática, fulminante
• 4. Propiltiuracilo: Es de ELECCIÓN en EMBARAZO O LACTANCIA
• 5. Agente de contraste yodados: ayudan en tirotoxicosis de cualquier etiología los cuales
inhibe la trimonoyodinacion de T4 por lo tanto aumenta T3 y disminuye tirotoxicosis
• 6. YODO RADIACTIVO: destruye tejido tiroideo sobre activo
• Que medidas se usan para complicaciones de enfermedad de Graves:
• 1. Oftalmopatia de Graves: EXOFTALMUS; ESTEROIDES, progresivo-radioterapia
• 2. Cardiacas: taquicardia sinusal; PROPANOLOL
• 3. Insuficiencia cardiaca: DIURÉTICOS Y DIGOXINA
• Que causa dermopatia de Graves?
• R = Engrosamiento de piel por glucosaminoglucanos, se le conoce como MIXEDEMA
PRETIBIAL Y SU TRATAMIENTO SON ESTEROIDES TÓPICOS.
PATOLOGIA TIROIDES
• Que caracteriza en la tirotoxicosis a la PARÁLISIS PERIÓDICA HIPOKALEMICA TIROTOXICA?
• R = PARÁLISIS FLÁCIDA SIMÉTRICA REPENTINA, HIPOKALEMIA E HIPOFOSFATEMIA. El
tratamiento es con PROPANOLOL que revierte la parálisis en 3 hrs. Dextrosa IV o CHBTS lo
agravan.
• Cual es el manejo de los nódulos tiroideos solitarios tóxicos?
• R = PROPANOLOL, YODO RADIACTIVO
• Cual es el manejo del bocio multinodular toxico?
• R = PROPANOLOL, YODO RADIACTIVO
• Cual es el cc de la TIROIDITIS DE HASHIMOTO O TIROIDITIS LINFOCITICA CRÓNICA y a que
enfermedad progresa?
1) Tiroides con AUMENTO DE TAMAÑO SIN DOLOR, de consistencia dura,
ASIMÉTRICO y fijo
2) PROGRESIÓN A HIPOTIROIDISMO.
3) Depresión, fatiga crónica, XEROSTOMÍA, XEROFTALMIA.
• Que anticuerpos son los principales implicados en la tiroiditis de Hashimoto?
• R = AC ANTIMIELOPEROXIDASA y LINFOCITOS TCD 4
PATOLOGIA DE TIROIDES
• Cual es el manejo de tiroiditis de Hashimoto al causar hipotiroidismo, bocio y Hashitoxicosis?
1) Hipotiroidismo: LEVOTIROXINA
2) Bocio: T4 para disminuir tamaño de glándula
3) Hashitoxicosis: Aumenta liberación de T4 por acumulo, PROPANOLOL o yodato sódico
• Cuales son los medicamentos principalmente vinculados con la tiroiditis inducida por fármacos?
• R = AMIODARONA, Interferon alfa, LITIO e interleucina 2.
• Cual es el cuadro clínico de la tiroiditis por fármacos?
• R = Tiroiditis INDOLORA
• Que caracteriza a la TIROIDITIS DE RIEDEL /tiroiditis fibrosa invasiva, tiroiditis leñosa, tiroiditis
lingeosa o tiroiditis invasiva?
1) Provoca HIPOTIROIDISMO e HIPOPARATIROIDISMO.
2) Se presenta en EDAD AVANZADA.
3) GLÁNDULA DURA CON ADHERENCIAS al cuello ocasionando DISNEA, DISFAGIA, DOLOR Y
RONQUERA.
4) Los trastornos incluyen FIBROSIS RETROPERITONEAL.
• Cuales son las características laboratoriales en la producción de T3 o T4 de distinción entre los
padecimientos causantes de hipertiroidismo?
1) TIROIDITIS SUB AGUDA (QUERVAIN)/ HASHIMOTO: >T4 + que T3
2) GRAVES/ BOCIO NODULAR TOXICO: >T3
PATOLOGIA DE TIROIDES
• Cual es la causa de la tiroiditis aguda o supurativa?
• R = Infecciosa
• Cual es el cuadro clínico de la tiroiditis aguda o supurativa?
• R = El paciente cursa con FIEBRE, disfagia, disfonía, DOLOR EN LA
REGIÓN ANTERIOR DEL CUELLO, eritema y GLANDULA TIROIDEA SENSIBLE
• Que muestra la biopsia en la tiroiditis aguda o supurativa?
• R = INFILTRADO LEUCOCITARIO
• Cual es el manejo de tiroiditis supurativa?
• R = Antibióticos + drenaje quirúrgico
• Cual es la etiología de la tiroiditis subaguda posparto?
• R = Es causada por la INFLAMACIÓN LINFOCITICA de la tiroides en los
primeros 3 A 6M POSTERIORES AL PARTO.
PATOLOGIA DE TIROIDES
• Cual es el cc de la tiroiditis subaguda posparto?
• R = EL BOCIO es INDOLORO
• Como se encuentra la VSG en la tiroiditis subaguda posparto?
• R = VSG NORMAL que la DIFERENCIA DE LA DE QUERVAIN.
• Cual es el manejo de la tiroiditis subaguda posparto?
1) Rara vez requiere tratamiento
2) En la FASE DE TIROTOXICOSIS es útil un BETABLOQUEADOR
3) En la fase de HIPOTIROIDISMO puede usarse LEVOTIROXINA
4) Los fármacos anti tiroideos y los esteroides NO ESTÁN INDICADOS
• Cual es la etiología de la tiroiditis subaguda o de Quervain?
1) Suele ser precedida por un PROCESO INFECCIOSO DE LAS VAS
2) Se ha relacionado con ECHOVIRUS, ADENOVIRUS, COXACKIE VIRUS Y VIRUS DE LA
PAROTIDITIS
3) La edad de presentación es entre 20-40ª
PATOLOGIA DE TIROIDES
• Cual es el cuadro clínico de la TIROIDITIS DE QUERVAIN?
EL cuadro clínico comienza con pródromos de :
1) MIALGIAS GENERALIZADAS
2) Faringitis
3) Fatigas
4) FIEBRE
5) DOLOR Y EDEMA DEL CUELLO
• Como se encuentra la VSG en la tiroiditis de Quervain?
1) ELEVACIÓN DE LA VSG
2) Elevación de PCR
• Cual es el manejo de tiroiditis subaguda o de Quervain?
• R = ASA es de elección, PROPANOLOL para SÍNTOMAS TIROTOXICOS
• Cual es el manejo del ESTROMA DE YEDEL?
• R = TAMOXIFENO de elección por años
PATOLOGIA DE TIROIDES
• Un teratoma ovárico que patología puede ocasionar, relacionado a la producción de
hormonas?
• R = TIROTOXICOSIS por producción de TSH
• Como afecta el aumento de hormonas tiroideas a los huesos?
• R = Aumenta la estimulación de la RESORCIÓN ÓSEA.
• Cuales son las manifestaciones clínicas del CRETINISMO?
• R = Alteración del desarrollo esquelético y del SNC, CARA TOSCA, PROTRUSIÓN DE LA
LENGUA Y HERNIA UMBILICAL.
• A que se le llama síndrome de plumer?
• R = Cuando uno de los nódulos del BOCIO MULTINODULAR se hace HIPERFUNCIONANTE
(bocio multinodular toxico), puesto que la mayoría de los pacientes son eutiroideos
• Porque en un paciente que utiliza corticosteroides de manera crónica se adelgaza y se ve
fina la piel?
• R = Debido al CATABOLISMO PROTEICO DE LAS PROTEÍNAS consistentes en perdidas de
colágeno y resorción ósea.
• Que alimentos se consideran bociogenos?
• R = Mandioca, col de brucelas, repollo y coliflor
PATOLOGIA DE TIROIDES
• Cual es la causa mas frecuente de bocio en el mundo?
• R = Déficit de yodo
• Que es el signo de PEMBERTON EN BOCIO RETROESTERNAL?
• R = Desfallecimiento, CONGESTIÓN FACIAL, OBSTRUCCIÓN VENOSA YUGULAR AL ELEVAR LOS
BRAZOS POR ENCIMA DE LA CABEZA.
• Que ocasiona el fenómeno de Jod-Basedow?
• R = En bocio multinodular, AL APLICAR SUSTANCIAS CON YODO, SE PUEDE PROVOCAR
HIPERFUNCIÓN DE ALGÚN NÓDULO.
• Cual es el tratamiento más eficaz para revertir el estado hipertiroideo de un bocio
multinodular toxico?
• R = YODO RADIACTIVO
• Cuales son los signos y síntomas en general del carcinoma tiroideo?
1) NÓDULO PALPABLE, INDOLORO ASINTOMÁTICO.
2) Estirpe Medular: BOCHORNOS, DIARREA Y SX DE CUSHING por producción de ACTH O
CRH.
• Que factores predisponen a carcinoma de tiroides?
• R = Proteína cinasa del protoncogen RET y ANTECEDENTES DE RADIACIÓN EN CABEZA Y
CUELLO.
PATOLOGIA DE TIROIDES
• Cual es la estirpe histológica mas común del carcinoma de tiroides y que lo caracteriza?
1) Carcinoma PAPILAR en 70-90%,
2) Asociado al GEN RET
3) El antecedente de RADIACIÓN EN CUELLO en la infancia se relaciona
4) Manifestándose como NÓDULO ÚNICO,
5) Donde la citología muestra CÉLULAS NUCLEARES SOBREPUESTAS CON APARIENCIA
DE VIDRIO ESMERILADO y surcos longitudinales.
6) Mejor pronostico,
• Con que enfermedades se relaciona el carcinoma papilar de tiroides?
1) Síndrome de GARDNER (poliposis adenomatosa familiar de colon)
2) Enfermedad de COWDEN (bocio familiar y hamartromas cutáneos)
3) Síndrome de TURCOT (tumores cerebrales mas poliposis en intestino grueso)
• A que tipo de cáncer tiroideo son mas propensos los pacientes con enfermedad de
Cowden?
1) FOLICULAR, AD,
2) Lo causa perdida de gen supresor de tumores.
3) Se caracteriza comúnmente por macrocefalia, HAMARTROMAS MÚLTIPLES, cáncer
mamario o PÓLIPOS INTESTINALES.
PATOLOGIA DE TIROIDES
• Que distingue al carcinoma FOLICULAR tiroideo del papilar?
1) Que ESTA ENCAPSULADO,
2) INVADE VASOS SANGUÍNEOS.
3) Su VARIEDAD extensamente INVASIVA son las CÉLULAS DE HURTLE
4) Tiene MAL PRONÓSTICO.
5) Al extirparlo se hace SEGUMIENTO CON TIROGLOBULINA para descartar metástasis
• Que origen tiene el CARCINOMA MEDULAR DE TIROIDES y que sustancia produce?
1) Se origina de las CÉLULAS C PARAFOLICULARES,
2) Secretor de CALCITONINA.
3) Secretor de ACTH
4) Asociado a mutación en oncogen RET, debido a eso SE DEBEN ESTUDIAR TAMBIÉN A
FAMILIARES
• Cual es el carcinoma de tiroides con peor pronostico?
1) ANAPLASICO con mortalidad del 95% en los primeros 6 meses.
2) Crecimiento rápido
3) Se han descrito supresión del gen supresor de tumores p53
PATOLOGIA TIROIDES
• Cual es el tratamiento de elección en carcinoma
tiroideo?
1) Extirpación quirúrgica
2) Tratamiento medico con la INHIBICIÓN DE LA
TSH DANDO LEVOTIROXINA A DOSIS ALTAS
3) Yodo radiactivo indicado en pacientes
POSTOPERADOS DE CA TIROIDEO
4) Radioterapia externa: Solo en pacientes con
EXTIRPACIÓN QUIRÚRGICA INCOMPLETA
5) Vigilancia clínica y por us
SX DE CUSHING
• Cual es la principal causa del Sx Cushing y su cuadro clínico?
1) Exógena por esteroides y 2da causa es por micro adenoma hipofisario
2) OBESIDAD CENTRAL
3) HAS por aumento del ANGIOTENSINOGENO HEPATICO
4) Intolerancia a la glucosa por incremento de la GLUCONEOGENESIS
inducida por los glucocorticoides
5) FACIES DE LUNA LLENA
6) Trastornos menstruales
7) HIRSUTISMO
• Como diagnosticas el sx de Cushing?
1) Inicialmente se debe demostrar HIPERCOLESTEROLISMO
2) Determinación de CORTISOL LIBRE URINARIO con sensibilidad de 90%
3) SUPRESIÓN CON DOSIS BAJAS DE DEXAMETASONA (prueba de
NUGENT)
4) Cuando un paciente tiene un cortisol urinario y serico a las 8:00 am > 1.8
después de 1 mg/dl de dexametasona a las 23 hrs de la noche previa, el
paciente tiene síndrome de Cushing.
SX DE CUSHING
• En cuanto al origen del síndrome de Cushing cuales son los estudios a realizar?
1) Se determina la hormona ADRENOCORTICOTROPICA plasmática, LOS VALORES POR
DEBAJO DEL LIMITE DE DETECCIÓN CATALOGAN AL SÍNDROME COMO DE ORIGEN
SUPRARRENAL.
2) La RM es la técnica de imagen de elección en el síndrome de Cushing de ORIGEN
HIPOFISARIO.
3) La TAC es de elección para visualizar las GLÁNDULAS SUPRARRENALES.
• En que consiste la PRUEBA DE NUGENT en el diagnostico de sx de Cushing?
• R = SUPRESIÓN DE ACTH con administrar 1mg de dexametasona a las 23 Hrs y con nueva
medición a las 8 AM.
• En que consiste el tratamiento medico del sx de Cushing?
• R = KETOCONAZOL, pues INHIBE LA SÍNTESIS DE CORTISOL pero se emplea por periodos
cortos pues es hepatotoxico.
• En que consiste el tratamiento quirúrgico del sx de Cushing?
1) CIRUGÍA HIPOFISARIA
2) RADIOTERAPIA HIPOFISARIA que ha mostrado ser efectiva en los casos que no se pueda
realizar la CIRUGÍA TRANSESFENOIDAL
ENFERMEDAD DE ADDISON
• Cual es la principal causa de la enfermedad de Addison/insuficiencia suprarrenal?
1) 90% adrenalitis AUTOINMUNE,
2) En países subdesarrollados TB.
• Cual es el cuadro clínico y DHE de la enfermedad de Addison?
1) HIPERPIGMENTACION de la piel (debido a que la deficiencia de cortisol,
provoca un amento compensatorio de la PROPIOMELANOCORTINA que es
precursora de la ACTH y de la hormona estimulante de melanocitos)
2) Vomito
3) Fatiga
4) Hipoglucemia
5) HIPOTENSIÓN
6) HIPONATREMIA
7) HIPERKALEMIA.
• Cual es el desequilibrio hidroelectrolitico principal en la enfermedad de Addison?
• R = ACIDOSIS METABOLICA con HIPERCALCEMIA.
ENFERMEDAD DE ADDISON
• Cuales son las manifestaciones clínicas de una CRISIS ADISONIANA o suprarrenal?
1) FIEBRE,
2) Deshidratación,
3) DOLOR ABDOMINAL
4) HIPOTENSIÓN O CHOQUE HIPOVOLEMICO,
5) Urea elevada
6) HIPONATREMIA,
7) HIPERKALEMIA,
8) HIPERCALCEMIA y
9) ACIDOSIS METABÓLICA.
• Cual es el tratamiento de la crisis Adisoniana?
• R = Administración inmediata de HIDROCORTISONA IV C-8
• Como realizas el diagnostico de enfermedad de Addison?
1) ESTANDAR DE ORO es la prueba de estrés con INSULINA IV la cual hace el diagnostico de adrenalitis
secundaria, DONDE EL CORTISOL DEBE AUMENTAR, SINO ENTONCES SE HACE EL DIAGNOSTICO.
2) Medición de CORTISOL DURANTE LA MAÑANA y ADRENOCORTICOTROPINA con NIVELES BAJOS.
3) ACTH IM y MEDIR CORTISOL A LOS 60 MIN, EL CUAL DEBE AUMENTAR.
4) En los pacientes con adrenalitis autoinmune se encuentran AUTOANTICUERPOS ANTI-21HIDROXILASA.
• Cual es el manejo de la enfermedad de Addison?
• R = Hidrocortisona o PREDNISONA.
SINDROME DE WATERHOUSE-
FRIEDERICHSEN
• Como se le llama al síndrome que sucede cuando una
HEMORRAGIA SUPRARRENAL MASIVA complica una
infección bacteriana y cual es el principal agente etiológico?
• R = Se le conoce como SINDROME DE WATERHOUSE-
FRIEDERICHSEN, frecuente en niños y su principal agente
etiológico es la SEPTICEMIA OCASIONADA POR NEISERIA
MENINGITIDIS
• Que consecuencias clínicas tiene el síndrome de
watherhouse-Friderichsen?
1) HIPOTENSION rápidamente progresiva que acaba en shock
2) CID con purpura generalizada
3) INSUFICIENCIA CORTICOSUPRARENAL
HIPERALDOSTERONISMO
• Cual es la etiología del hiperaldosteronismo?
1) Primario: Hipersecreción de aldosterona la cual NO SE SUPRIME EN RESPUESTA A LA EXPANSIÓN DE
VOLUMEN.
2) Secundario: Altos niveles de renina la cual se reprime en respuesta a la expansión de volumen.
• Cuales son los datos clínicos del hiperaldosteronismo?
1) Puede cursar asintomático o
2) Cefalea
3) HAS
4) DEBILIDAD MUSCULAR
5) PARESTESIAS
6) Tetania
7) Parálisis
• Cuales son los datos de laboratorio encontrados en Hiperaldosteronismo primario?
• R = HIPERNATREMIA, HIPOKALEMIA y ALCALOSIS METABÓLICA
• -A 30-year-old man is having symtoms of muscle weakness, fatigue, and headaches. On examination, his blood
pressure is 180/100 mm Hg, pulse 80/min, JVP 4 cm, heart sounds normal, and lungs clear. His serum
potassium level is 2.5 mEq/L and bicarbonate 30 mEq/L. The urine potassium is 40 mEq/L. Plasma renin is low
and aldosterone is high.
• -Primary hyperaldosteronism is characterized by hypertension with high plasma aldosterone and low plasma
rennin
HIPERALDOSTERONISMO
• Como diagnosticas hiperaldosteronismo?
• R = ELEVACIÓN DE ALDOSTERONA PLASMÁTICA POSTERIOR A LA INGESTIÓN DE
CLORURO DE SODIO por cada comida durante dos a tres días o un NIVEL DE
ALDOSTERONA EN PLASMA ALTO DESPUÉS DE LA INFUSIÓN INTRAVENOSA DE
SOLUCIÓN SALINA (1 lt x hr durante 4 hrs).
• Cual es el manejo del hiperaldosteronismo?
1) CIRUGÍA A PACIENTES CON ALDOSTERONOMA con adrenalectomía laparoscópica
2) En el caso de HIPERPLASIA SUPRARRENAL BILATERAL IDIOPÁTICA se debe recibir
TRATAMIENTO MEDICO CON ESPIRONOLACTONA
3) En el caso de hiperaldosteronismo tratable con esteroides se utiliza la
DEXAMETASONA para mantener una supresión del eje hipotálamo hipófisis.
• Cuales son los datos que caracterizan al síndrome de Conn en
hiperaldosteronismo?
• R = Hipotensión, hipokalemia, aumento de aldosterona y supresión secundaria de
renina.
HIPOALDOSTERONISMO
• Cual es la etiología del hipoaldosteronismo?
1) Puede producirse por un estimulo insuficiente por parte de la renina
(HIPORRENINEMICO) “NEFROPATÍA DIABÉTICA”
2) Por un fallo primario en la secreción de aldosterona (HIPERRENINEMICO)
“ENFERMEDAD DE ADDISON”
3) Por resistencia a la acción de la aldosterona
(SEUDOHIPOALDOSTERONISMO).
4) Algunos medicamentos pueden ocasionarlo como los AINES, heparina (suprime la
secreción de renina), IECAS.
• Cual es el cuadro clínico del hipoaldosteronismo?
1) CALAMBRES
2) Debilidad muscular
• Cuales son los datos de laboratorio del hipoaldosteronismo?
1) HIPERKALEMIA
2) HIPONATREMIA
3) ACIDOSIS METABOLICA HIPERCLOREMICA
4) Deshidratación
5) Se debe hacer diagnostico diferencial de enfermedad de Addison
HIPOALDOSTERONISMO
• Como se diagnostica el hipoaldosteronismo?
1) Analíticamente se deben obtener los niveles plasmáticos de aldosterona,
cortisol y actividad de renina
2) Si se sospecha de enfermedad de ADDISON SE DEBE REALIZAR LA PRUEBA
DE ESTIMULACIÓN CON HORMONA ACTH.
3) Si existe aldosterona baja con actividad de renina (HIPORRENINEMICO),
debemos pensar en NEFROPATIA DIABETICA,
4) Si obtenemos aldosterona baja y renina alta (HIPERRENINEMICO),
debemos sospechar ADISON o una forma grave de hiperplasia suprarrenal
congénita.
5) Si lo que nos reportan es ALDOSTERONA ELEVADA Y ACTIVIDAD DE
RENINA ALTA, debemos pensar en SEUDOHIPOALDOSTERONISMO.
• Cual es el manejo del hipoaldosteronismo
1) INGESTIÓN LIBRE DE SAL como TRATAMIENTO MEDICO no farmacológico
2) El tratamiento medico CON FÁRMACOS es con FLUDROCORTISONA.
ALTERACIONES POR DHE, Na y K
• ENFERMEDAD DE ADDISON:
• Acidosis Metabolica con hipercalcemia
• DHE: Hiponatremia – Hiperkalemia
• Cual es el manejo de la enfermedad de Addison?
• R = Hidrocortisona o PREDNISONA.
• HIPOALDOSTERONISMO:
• Acidosis Metabolica Hipercloremica
• DHE: Hiponatremia – Hiperkalemia
• Cual es el manejo del hipoaldosteronismo
1) INGESTIÓN LIBRE DE SAL como TRATAMIENTO MEDICO no farmacológico
2) El tratamiento medico CON FÁRMACOS es con FLUDROCORTISONA.
• HIPERALDOSTERONISMO:
• Alcalosis Metabolica
• DHE: Hipernatremia - Hipokalemia
• Cual es el manejo del hiperaldosteronismo?
1) CIRUGÍA A PACIENTES CON ALDOSTERONOMA con adrenalectomía laparoscópica
2) En el caso de HIPERPLASIA SUPRARRENAL BILATERAL IDIOPÁTICA se debe recibir
TRATAMIENTO MEDICO CON ESPIRONOLACTONA
3) En el caso de hiperaldosteronismo tratable con esteroides se utiliza la DEXAMETASONA
para mantener una supresión del eje hipotálamo hipófisis.
HIPERCALCEMIA
• Cuales son los eventos metabólicos implicados en la regulación del calcio?
1) En presencia de hipercalcemia se estimula la secreción de CALCITONINA, la cual INHIBE LA OSTEOLISIS y
ESTIMULA LA CALCIURIA.
2) Cuando la hipercalcemia permanece, la CALCITONINA INHIBE LA REABSORCIÓN OSTEOCLASTICA, LA
SECRECIÓN DE PARATHORMONA Y LA SÍNTESIS DE VITAMINA D.
• Cual es la etiología de la hipercalcemia?
1) HIPERPARATIROIDISMO PRIMARIO: Puede deberse a ADENOMA ÚNICO o HIPERTROFIA GLANDULAR. Se
caracteriza por ELEVACIÓN DE LA PTH, HIPERCALCEMIA E HIPOFOSFATEMIA.
2) Hiperparatiroidismo secundario: Puede deberse a IR, sx de mal absorción y raquitismo.
3) El adenoma y las neoplasias malignas causan 90% de las hipercalcemias.
4) Es frecuente en el CÁNCER DE MAMA.
• Cual es el cuadro clínico de la hipercalcemia?
1) Ca entre 10.5-12 g/dL es asintomático
2) Niveles mayores forman TRIADA CLÁSICA DE ESTREÑIMIENTO, NAUSEA Y VOMITO.
3) Fatiga, letargo, cefalea, sed, nausea, vomito, NEFROLITIASIS,
4) ACORTAMIENTO DEL INTERVALO QT EN EL EKG.
• Cual es el manejo de la hipercalcemia?
1) HIDRATACION
2) FUROSEMIDE es eficaz para evitar la reabsorción tubular de calcio
• Cual es el fármaco que se indica en hipercalcemia crónica?
• R = TIAZIDAS pues previenen cálculos renales.
HIPOCALCEMIA
• Cual es la etiología de la hipocalcemia?
1) Ca sérico menor 8.5. Previamente se debe confirmar que la albumina se encuentre en niveles séricos
normales, ya que el descenso de 1 g/dL de albumina se acompaña de un descenso de 0.8 mg/dL de
calcio.
2) La causa mas común de HIPOPARATIROIDISMO ES LA CIRUGÍA TIROIDEA Y LA SEGUNDA ES LA
RADIACIÓN.
3) La hipomagnesemia < 1 mg/dL puede producir hipocalcemia
4) La hipermagnesmia reduce la secreción de PTH.
5) La hiperfosfatemia causa hipercalcemia por formación de complejos con el calcio.
6) En pacientes transfundidos se produce hipocalcemia ya que el citrato del paquete globular actúa como
paquete globular.
7) EL SINDROME DE DIGEORGE se caracteriza por AUSENCIA DE GLÁNDULAS PARATORIDEAS, con DÉFICIT
DE PTH, ASOCIADO A DISPLASIA TIMICA, y malformaciones cardiacas por alteración en el desarrollo
de LAS TERCERAS Y CUARTAS BOLSAS FARÍNGEAS.
8) Las formas adultas de hipoparatiroidismo se asocian a enfermedades autoinmunes como insuficiencia
suprarrenal, anemia perniciosa e hipotiroidismo.
9) En la PANCREATITIS, el calcio se deposita en áreas con necrosis grasa.
10) EL SINDROME DE HUESO HAMBRIENTO se presenta en paratiroidectomia.
11) En enfermedades del APARATO DIGESTIVO se favorece el déficit de vitamina D por disminución en la
absorción.
12) La DIFENILHIDANTOINA inhibe la absorción de calcio.
• Cual es el cuadro clínico de la hipocalcemia?
1) HIPEREXITABILIDAD neuromuscular.
2) Parestesias, HIPERREFLEXIA, ESPASMO CARPOPEDAL, irritavilidad, SIGNO DE CHEVOSTEK Y TROSSEAU.
3) El EKG MUESTRA PROLONGACIÓN DEL ST.
OSTEOPOROSIS
• Cual es el manejo de la osteoporosis?
1) ALENDRONATO, RISENDRONATO.
2) PTH a dosis bajas, al igual que los bifosfonatos han demostrado aumento en la
masa ósea,
3) Terapia de reemplazo hormonal con estrógenos previene fracturas vertebrales.
4) RALOXIFENO Y TAMOXIFENO ya que AUMENTAN LA DENSIDAD ÓSEA,
DISMINUYEN RIESGO DE FRACTURAS VERTEBRALES Y EL CÁNCER DE MAMA
5) La TIBOLONA es una sustancia sintética con acción tejido específica que tiene
efecto ESTROGÉNICO, PROGESTACIONAL Y ANDROGÉNICO según el lugar de
acción. Esta molécula mejora los síntomas vasomotores, el estado anímico, la
libido, la atrofia urogenital y previene la osteoporosis postmenopáusica; no
produce mastalgia ni modifica la densidad mamográfica. Se observan además
efectos beneficiosos sobre algunos parámetros cardiovasculares y hemostáticos.
EN EL ENDOMETRIO, LA TIBOLONA SE CONVIERTE EN SU METABOLITO
PROGESTAGÉNICO/ANDROGÉNICO Y EN CONSECUENCIA NO PRODUCE
ESTIMULACIÓN ENDOMETRIAL NI SE NECESITA AGREGAR UN PROGESTÁGENO.
Esta ventaja mejora el cumplimiento de la terapia hormonal de reemplazo (THR)
debido que no provoca sangrados por disrupción
DENSITOMETRIA OSEA
OSTEOMALACIA
• Cual es la etiología de la osteomalacia?
• R = Déficit de VITAMINA D EN ADULTOS
• Cual es el cuadro clínico de la osteomalacia?
1. DEBILIDAD MUSCULAR PROXIMAL, asociada con desgaste muscular, hipotonía y dificultad para la
movilización.
2. EL OSTEOCOPO (dolor óseo), es mas notorio en LA REGION LUMBAR, PELVIS Y EXTREMIDADES
PELVICAS DONDE SE PRESENTAN FRACTURAS.
3. Las fracturas suceden con un mínimo de traumatismo.
4. Los síntomas deben incluir: DOLOR OSEO, DEFORMIDADES DEL ESQUELETO Y/O CRANEO, piernas
arqueadas o rodillas malformadas, PECHO EN PALOMA, Deformaciones dentales.
• Como se diagnostica la osteomalacia?
1. MARCLAJE CON TETRACICLINAS. Este medicamento se deposita en forma de bandas en el sitio de
mineralización y al ser fluorescentes pueden ser visualizados al microscopio. La tasa de crecimiento
del esqueleto puede estimarse en las biopsias de las crestas iliacas al medir la distancia entre las
bandas de tetraciclina. Si disminuye la distancia entre las bandas se hace el diagnostico.
2. EL HALLAZGO RADIOGRÁFICO CARACTERÍSTICO ES LA REDUCCIÓN EN LA
DENSIDAD ÓSEA CON ADELGAZAMIENTO DE LA CORTEZA.
3. Laboratorio: Los pacientes con déficit de vitamina D se presentan con HIPOFOSFATEMIA, CALCIO
SÉRICO BAJO E HIPOPARATIROIDISMO SECUNDARIO.
• Cual es el manejo de la osteomalacia?
• R = Calcitriol o CALCIO + VITAMINA D en combinación.
DEFICIT DE ADH
• Que causa la deficiencia de ADH?
• R = Diabetes insípida central con POLIURIA Y POLIDIPSIA, causando
HIPONATREMIA con la ingesta inadecuada de líquidos
• -A 27-year-old woman presents because she is feeling unwell. Her
examination is normal, but her serum sodium is 115 mEq/L. Her urine lytes:
sodium 55 mEq/L, osmolality 100 mOsm/L. This condition causes the
greatest amount of medullary washout:
• -Primary áternn can cause greatermedullary washout át either nephrogenic
or central DI because primary áternn tends to cause áternn of the ECF
átern. This tends to increase total delivery of sodium chloride and áter to
the inner medulla. It also increases renal blood flow, and increased flow
through the vasa recta reduces ability to trap solutes in the medulla.
DIABETES INSIPIDA CENTRAL
• A que se debe la diabetes insípida central primaria y secundaria?
1) Primaria : Ac vs ARGININA-VASOPRESINA.
2) LESIÓN DEL HIPOTÁLAMO, del tallo hipofisario o por cualquier CAUSA
INFECCIOSA, TRAUMÁTICA etc..
• Cuales son las causas de DI central y cuales son los hallazgos en la TAC?
• R = DI IS USUALLY CAUSED BY DESTRUCTION, OR AGENESIS, OF THE POSTERIOR
PITUITARY, its normal signaling is lost. PITUITARY DI CAN ALSO RESULT FROM
TRAUMA, TUMORS (BOTH PRIMARY AND SECONDARY), granulomas, infections,
inflammatory diseases, chemical toxins, congenital malformations, and genetic
disorders. Depending on the cause, the MRI may demonstrate other associated
findings.
• Que causa diabetes insípida en el ultimo trimestre de embarazo-puerperio?
• R = Es inducido por VASOPRESINA en el ultimo trimestre de embarazo, relacionada
frecuentemente con oligohidramnios, preeclampsia o disfunción hepática.
RESPONDE A DESMOPRESINA
DIABETES INSIPIDA NEFROGENICA
• Que causa y cual es el cc de la diabetes insípida nefrogenica?
1) DEFECTO TUBULORENAL DE LOS RECEPTORES DE LA ADH que interfiere con la reabsorción
de agua.
2) POLIURIA QUE NO RESPONDE A VASOPRESINA, con secreción normal vasopresina.
3) Congénita: Desde el nacimiento y se debe a expresión defectuosa de los RECEPTORES
RENALES DE VASOPRESINA V2.
• -A 38-year-old man is admitted to hospital for investigation of polyuria and a low serum
sodium concentration. After a fluid deprivation test, there was no change in his urine
osmolality. He was then given vasopressin (DDAVP) with no change in urine volume or urine
osmolality after 2 hours.
• -There is little or no response to vasopressin after fluid deprivation in complete nephrogenic
DI. Incomplete nephrogenic DI will show some response.
• -A19-year-old man and one of his two brothers has had polyuria and polydipsia since birth.
Neither his sisters nor his parents are affected.
• -Nephrogenic DI can be inherited on the X chromosome. Its X-linked recessive nature means
that males are predominantly affected. Only women who are homozygous are affected.
DIABETES INSIPIDA NEFROGENICA
• Como diagnosticas DM insípida?
1) JUICIO CLÍNICO, no hay prueba de laboratorio contundente.
2) RECOLECCIÓN DE ORINA DE 24 HRS MIDIENDO VOLUMEN <2LTS/24 HRS SIN
HIPERNATREMIA.
3) Desmopresina: MIDIENDO VOLUMEN URINARIO 12 HRS ANTES Y 12 HRS DESPUÉS, EN EL
CUAL LOS PACIENTES CON DIABETES INSÍPIDA CENTRAL HABRÁ DISMINUCIÓN DE SED Y
POLIURIA,
4) Realizándose IRM analizando hipófisis e hipotálamo, se observa saco hipofisario
engrosado: Células de langerhans, sarcoidosis, hipofisitis linfocitica, Histiocitosis
(proliferación de macrófagos en diferentes aparatos y sistemas).
5) Desmopresina en DM nefrogenica valorando su concentración con restricción de líquidos
aumentando sus valores.
• Cual es el manejo de la DM insípida nefrogenica o central?
1) CENTRAL: DESMOPRESINA tmb se utiliza en DM INSÍPIDA RELACIONADA CON
EMBARAZO o puerperio, evitar factores de agravamiento como administración de
glucocorticoides.
2) NEFROGENICA: HIDROCLOROTIAZIDA con suplemento de potasio, INDOMETACINA en
fase aguda.
ACROMEGALIA• Cual es la causa principal de acromegalia?
• R = ADENOMA HIPOFISIARIO, también por secreción ectópica de GnRH o GH por tumores carcinoides hipotalámicos,
bronquiales o pancreáticos.
• Que características forman parte del SÍNDROME DE CARNEY?
• R = MIXOMA AURICULAR, NEUROMA DEL ACÚSTICO Y PIGMENTACIÓN PUNTEADA DE LA PIEL
• Cuales son los signos y síntomas de la acromegalia o gigantismo?
1) ESTATURA ELEVADA, gigantismo antes del cierre hiposifiario.
2) Acromegalia significa crecimiento de extremidades.
3) SÍNDROME DEL TÚNEL CARPIANO.
4) RASGOS FACIALES TOSCOS, PROGNATISMO CON MALA OCLUSIÓN.
5) HAS 50%. DM 30%
• Cuales son los lab para dx acromegalia o gigantismo?
1) AYUNO. > IGF-1 5 VECES DE LO NORMAL.
2) Prolactina aumentada algunas veces.
3) Glucosa: la DM ES COMÚN.
4) Calcio: Para descartar hiperparatiroidismo.
5) Fosforo: Se encuentra aumentada.
6) T4 y TSH: HIPOTIROIDISMO COMÚN.
7) SE RECOMIENDA ADMINISTRAR UNA SOLUCIÓN DE DEXTROSA MIDIENDO LA GH 1 HR DESPUÉS, SI LOS VALORES ESTÁN < O
NORMALES AL IGUAL QUE LA IGF-1, YA QUE ESTA HORMONA RESPONDE CON DISMINUCIÓN A LA
INGESTA DE GLUCOSA.
• Que medicamentos se utilizan en caso de no remitir > GH o IGF-1 con adenectomía transesfenoidal hipofisiaria?
1) ANÁLOGOS DE LA SOMATOSTATINA COMO OCTREOTIDE para conservar disminución del tamaño del adenoma. GH sérica 1
mg y 2.5 mg, pero no responde bien si previos al tratamiento la GH > 20 ng/ml.
2) PEGVISOMANT es un antagonista del receptor de la GH bloqueando sus efectos, vía SC, pero no reduce el tamaño de los
adenomas.
3) RADIOCIRUGÍA: en los pacientes no responden al tratamiento quirúrgico o medico, no se realiza si hay extensión suprecelar
por que puede dañar el quiasma óptico.
NEM I
• Cuales son las características de la NEM I O SÍNDROME DE WERMER?
1) Se caracteriza por neoplasias en PARATIROIDES, PÁNCREAS y ADENOHIPOFISIS.
2) Se hereda de forma AD en el CROMOSOMA 11
• Cual es la clínica del NEM I o Sindrome de Wermer?
1) HIPERPARATIROIDISMO PRIMARIO: Que puede manifestarse con MIALGIAS, DEBILIDAD, HIPERCALCEMIA,
ALTERACIONES MENTALES, NEFROLITIASIS Y ANORMALIDADES ÓSEAS. La hipercalcemia puede provocar
aumento de la secreción de gastrina y un Sx Zollinger-Ellison secundario. EL TRATAMIENTO CONSISTE EN LA
EXTIRPACIÓN QUIRÚRGICA DE LAS PARATIROIDES.
2) TUMORES DE LAS CÉLULAS DE LOS ISLOTES PANCREÁTICOS: Los GASTRINOMAS SON LA CAUSA MAS
IMPORTANTE de morbimortalidad y la mayoría son multiples. Los insulinomas son generalmente benignos y
únicos.
3) NEOPLASIA DE LA ADENOHIPOFISIS: La neoplasia MAS FRECUENTE ES EL PROLACTINOMA seguida del tumor
secretor de hormona del crecimiento que causa acromegalia, pueden presentarse como enfermedad de
Cushing secundaria a adenoma hipofisario.
• Como realizas el diagnostico de NEM I?
1) Con 2 de las 3 neoplasias características del síndrome
2) El MEN I familiar se define como la presencia de un paciente afectado con esta enfermedad y un familiar de
primer grado que presente al menos una de las tres neoplasias características
• Cual es el manejo del NEM I?
1) CIRUGÍA
2) PARA CADA MANIFESTACIÓN de sobre producción hormonal se da el medicamento individual
MEN II
• Cual es la etiología cromosómica del MEN II?
• R = Se hereda de forma AD, en el CROMOSOMA 10.
• Cual es la clasificación del MEN II?
 MEN II A o Síndrome de Sipple: Con la presencia de cáncer medular de
tiroides, FEOCROMOCITOMA e hiperparatiroidismo primario.
 MEN II B: Con la presencia de cáncer medular de tiroides, feocromocitoma,
NEUROMAS MUCOSOS, ganglioneuromatosis intestinal y HABITO
MARFANOIDE.
• Como diagnosticas el MEN II?
1) 2 a 3 tumores de los antes mencionados
2) La mutación del protoncogen RET esta indicada en todos los familiares de
primer grado.
3) EL CARCINOMA MEDULAR DE TIROIDES SUELE SER LA PRINCIPAL CAUSA
DE MUERTE.
GASTRINOMA
• Cuales son los tumores de las células de los islotes pancreáticos?
• R = Gastrinoma, insulinoma, glucagonoma, vipoma y somatostinoma. GIGVS
• Cual es el cuadro clínico del gastrinoma?
1) Enfermedad ulcerosa con dolor abdominal
2) Diarrea
3) ERGE.
4) Se debe sospechar en ULCERAS PÉPTICAS MÚLTIPLES.
• Como diagnosticas el gastrinoma?
1) Los niveles séricos de GASTRINA EN AYUNO MAYORES DE 1000 Y PH < 2.5 EN LA SECRECIÓN GÁSTRICA
HACEN EL DIAGNOSTICO DE GASTRINOMA.
2) Se debe SUSPENDER LA ADMINISTRACIÓN DE ANTAGONISTAS DE H2 UNA SEMANA ANTES Y DE
OMEPRAZOL.
3) En paciente con niveles séricos menores a 1000 y PH < 2.5 se debe pensar en H. Pilory
4) La aclorhidria por gastritis atrófica o anemia perniciosa es una de las causas mas frecuentes de
hipergastrinemia.
• Cual es el manejo del gastrinoma?
1) Para la hipersecreción acida OMEPRAZOL
2) Gastrinoma RESECCIÓN QUIRÚRGICA PARA EVITAR METÁSTASIS
INSULINOMA• Cual es el cuadro clínico del insulinoma?
1) SECUNDARIOS AL EFECTO HIPOGLUCEMICO como cefalea, mareo, debilidad, confusión, desorientación y coma.
2) Síntomas secundarios al aumento de catecolaminas circulantes provocado por la hipoglucemia como sudación, temblor y
palpitación.
3) La mayoría de las crisis se relaciona con el ayuno.
• Como realizas el diagnostico de insulinoma?
1) Se basa en la TRIADA DE WIPPLE la que consiste en síntomas de neuroglucopenia, niveles de glucosa en sangre por debajo
de 45 y mejoría inmediata de los síntomas después de la administración de glucosa
2) Si se sospecha de insulinoma se debe realizar una PRUEBA DE AYUNO DURANTE 72 HRS, considerado como el método
diagnostico de elección, donde se miden los niveles de glucosa, péptido C e insulina cada 6 hrs y siempre que exista clínica de
hipoglucemia la prueba se suspende si aparece una glucemia inferior a 45 o si se presentan síntomas neuroglucopenicos. En
sujetos normales la insulina debe disminuir en presencia de hipoglucemia. Los criterios diagnósticos de insulinoma incluyen
insulina plasmática mayor de 6 cuando la glucemia es menor de 40, proinsulina > .2, péptido C > 200 y cociente de
insulina/glucosa > .3.
3) Existen AINTICUERPOS ANTIINSULINA Y NIVELES ELEVEADOS DE PROINSULINA.
• Cual es el manejo del insulinoma?
1) Para las crisis de hipoglucemia se realizan comidas frecuentes y de escasa cantidad
2) En el caso de hipoglucemia se administra 50 ml de dextrosa al 50%, si no hay respuesta dentro de los primeros 15 minutos, la
dosis puede repetirse, en caso de que no se pueda canalizar al paciente se administra 1 mg de guagón por vía IV, IM o
subcutánea.
3) El DIAZOXIDO inhibe la secreción de insulina, pero provoca retención de líquidos e hirsutismo con frecuencia.
4) La resección completa del tumor cura a la mayoría de los pacientes. Cuando no se logra localizar el tumor se realiza una
pancreatectomia distal gradual hasta localizarlo dejando por lo menos 20-30% del páncreas hasta localizarlo.
5) EN EL CASO DE TUMOR MALIGNO CON METÁSTASIS O TUMOR IRRESECABLE SE RECOMIENDA EL
TRATAMIENTO A LARGO PLAZO CON DIAZOXIDO, SI NO RESPONDE SE INICIA QUIMIOEMBOLIZACION.
VIPOMAS• Que son los vipomas?
• R = Son tumores derivados de las CÉLULAS DE LOS ISLOTES DEL PÁNCREAS que producen grandes
cantidades de PEPTIDO INTESTINAL VASOACTIVO, el cual AUMENTA LA CONTRACCIÓN
DEL MUSCULO LISO, INHIBE LA SECRESION GÁSTRICA Y TIENE EFECTOS VASODILATADORES.
• Cual es el cuadro clínico asociado a vipomas?
1) El síntoma predominante es una DIARREA ACUOSA, que produce deshidratación,
hipokalemia y debilidad.
2) La DIARREA ES SECRETORA, persiste durante el ayuno y NO RESPONDE A
ANTIDIARREICOS.
3) Acidosis metabólica
• Como realizas el diagnostico del vipoma?
1) Presencia de DIARREA SECRETORA
2) Niveles elevado de PÉPTIDO INTESTINAL VASOACTIVO
3) TUMORACION pancreática
• Como se maneja el vipoma?
1) Corregir la deshidratación y desequilibrio hidroelectrolitico, en especial de la HIPOKALEMIA.
2) Resección del tumor y en presencia de metástasis se reduce primero el tumor.
3) Si no se encuentra el tumor se busca en las glándulas suprarrenales y las cadenas simpáticas, en caso
de no identificarse en ninguno de estos sitios se realiza resección del 80% del tumor.
4) En el caso de tumor maligno inoperable o presencia de metástasis se recomienda el uso del análogo
de la somatostatina de acción prolongada, también puede utilizarse la embolizacion de la arteria
hepática.
GLUCAGONOMAS
• Que son los glucagonomas?
1) Son tumores derivados de las CELULAS ALFA DE LOS ISLOTES DEL PANCREAS los cuales producen una gran
cantidad de la HORMONA GLUCAGON y puede secretar otros péptidos (polipeptido pancreático, gastrina y somatostatina).
2) Es mas frecuente en mujeres PERI O POSTMENOPAUSICAS
• Cual es el cuadro clínico de los glucagonomas?
1) DERMATITIS NECROLITICA MIGRATORIA es característica de los glucagonomas. Inicia como una zona eritematosa y
escamosa en la ingle y glúteo, se extiende al perineo y extremidades inferiores, en ocasiones hay lesiones peri orales,
posteriormente se hace ampolloso y deja erosiones cuando las ampollas se rompen.
2) Intolerancia a la glucosa o DIABETES
3) Anemia
4) QUEILITIS Y ESTOMATITIS
5) Diarrea
6) Hipercolesterolemia
• Como diagnosticas los glucagonomas?
1) Con el cuadro clínico y se confirma al encontrar niveles plasmáticos de GLUCAGON EN AYUNO > 1000 (NORMAL 150) Y
UNA TUMORACIÓN PANCREÁTICA.
2) NO SE SUPRIME LA LIBERACIÓN DEL GLUCAGON CON LA ADMINISTRACIÓN DE GLUCOSA Y TAMPOCO SE
ESTIMULA CON LA ADMINISTRACIÓN DE ARGININA.
• Cual es el manejo del glucagonoma?
1) Debido a la presencia de METASTASIS EN LA MAYORIA DE LOS PACIENTES, no es posible una resección quirúrgica
curativa, pero en estos casos se extirpa el tumor primario y se reduce el volumen de las metástasis.
2) Los SUPLEMENTOS DE ZINC mejoran la dermatitis
3) Los ANALOGOS DE LA SOMATOSTATINA normalizan los valores de glucagon y aminoácidos séricos, mejoran la dermatitis
y la diarrea, promueven el aumento de peso, pero NO corrigen la intolerancia a la glucosa.
4) La quimioterapia NO es útil pero cuando se decide utilizarla, la combinación que ha demostrado mas eficacia es la de
ESTREPTOZOCINA +DACARBACINA.
5) EL 50% DE LAS MUERTES ES CAUSADO POR EVENTOS TROMBOEMBOLICOS.
SOMATOSTINOMAS
• Que son los somatostatinomas?
1) Son tumores derivados de las CELULAS DELTA DEL PANCREAS productores de
somatostatina, al igual que los VIPomas y glucagonomas
2) 70% SON MALIGNOS
3) La somatostatina es un inhibidor potente de la secreción gástrica y pancreática
• Cual es el cuadro clínico de los somatostatinomas?
1) Es causado por la inhibición de la secreción tanto exocrina como endocrina del páncreas
2) LA TRIADA CLASICA CONSISTE EN DIABETES, DIARREA-ESTEATORREA Y
COLELITIASIS.
• Como diagnosticas los somatostatinomas?
• R = La mayoría se localiza incidentalmente al realizar cirugías abdominales o endoscopias. EL
diagnostico se realiza al demostrar la somatostatina elevada en plasma.
• Cual es el manejo de los somatostatinamas?
1) ANÁLOGOS DE LA SOMATOSTATINA con el fin de inhibir la activación de los receptores por
parte de la somatostatina secretada por el tumor.
2) EXTIRPACIÓN QUIRÚRGICA si se localiza el tumor
3) Para enfermedad metastasica se usa QUIMIOTERAPIA
FEOCROMOCITOMA
• Cual es la etiología del feocromocitoma?
1) Tumores de origen NEUROECTODERMICO desarrollado a partir de las CÉLULAS CROMAFINES DEL SNS (ganglios
simpáticos y medula suprarrenal) productores de CATECOLAMINAS y la mayoría de ellos libera epinefrina y
norepinefrina.
2) Los feocromocitomas ASOCIADOS A NEM SECRETAN SOLO EPINEFRINA.
• Cual es la clasificación y localización mas común de los feocromocitomas?
1) Feocromocitomas si provienen de las células cromafines y paragangliomas si son de origen extra renal.
2) La localización mas frecuente es la adrenal 85%, siempre es unilateral, en caso de ser bilateral debe
sospecharse de NEM
• Cual es el cuadro clínico del feocromocitoma?
1) HAS 90%
2) Crisis adrenal caracterizada por triada clásica de: CEFALEA, PALPITACIONES Y SUDORACIÓN.
• Cuales son los métodos diagnósticos de feocromocitoma?
1) Demostración de niveles elevados de catecolaminas o sus metabolitos en sangre o en orina, lo que establece el
diagnostico.
2) Se determina el nivel de catecolamina libre o no metabolizadas (epinefrina y norepinefrina) o sus metabolitos (
metanefrina, normetanefrina y ACIDO VANILMANDELICO) en orina de 24 hrs.
• Cuales son los métodos de imagen para dx de feocromocitoma?
1) TAC
2) Gamagrafia para localizar paragangliomas
• Cual es el manejo del feocromocitoma?
1) Cirugía, pero antes de realizarla es necesario realizar un bloqueo alfadrenergico, generalmente se utiliza
FENOXIBENZAMINA durante 10-14d previos a la cirugía.
2) Durante la anestesia o manipulación del tumor SUELE APARECER HIPERTENSIÓN Y ARRITMIAS, LA PRIMERA SE
TRATA CON FENTOLAMINA Y LA SEGUNDA CON PROPANOLOL.
TUMORES CARCINOIDES
• Cual es la etiología de los tumores carcinoides?
• R = Provienen de las CELULAS NEUROENDOCRINAS, en su
citoplasma hay gránulos llenos de hormonas (serotonina, histamina,
dopamina, prostaglandinas.
• Desde su origen embriológico, donde se localizan los carcinoides
del intestino anterior, medio y posterior?
1) Anterior: Pulmones, bronquios y estomago
2) Medio: Intestino delgado, apéndice o colon proximal.
3) Posterior: Colon distal y recto.
• Cual es la localización más común del tumor carcinoide?
• R = APÉNDICE, luego recto, íleon, pulmón, bronquios y estomago.
TUMOR CARCINOIDE
• Cual es el cuadro clínico producido por el tumor carcinoide?
• R = Producen síntomas inespecíficos, vagos, relacionados con el órgano afectado tales como:
1) Carcinoide apendicular: El la LOCALIZACIÓN MAS FRECUENTE. Si el tumor mide mas de 2 cm se debe realizar una
hemicolectomia derecha, si mide entre 1-2 cm e invade la serosa se debe realizar una hemicolectomia derecha y
linfadenectomia regional. LA APENDICETOMÍA SE CONSIDERA CURATIVA CUANDO MIDEN MENOS DE 1 CM.
2) Carcinoide rectal: Estos secretan mas glucagon mas que serotonina. Puede manifestarse como SANGRADO RECTAL,
DOLOR O CONSTIPACION. Los tumores mayores de > 2 cm se tratan con una resección anterior baja o una
abdominoperineal.
3) Carcinoide del intestino delgado: La mayoría se encuentra el ILEON. Pueden provocar desde molestias abdominales
leves hasta dolor abdominal intermitente causado por la obstrucción intestinal, la cual puede ser secundaria a la
fibrosis que producen estos tumores o con menor frecuencia se manifiestan con diarrea, hemorragia por ulceración
de la mucosa o intususcepción. SON LA CAUSA MAS FRECEUNTE DE SX CARCINOIDE.
4) Carcinoides pulmonares y bronquiales: Neumonías recurrentes, tos hemoptisis, dolor pleurítico. La resección en
cuna o segmentaria se recomienda para los tumores localizados en caso de los carcinoides atípicos se utilizan
resecciones mas amplias.
5) Carcinoides gástricos: Se originan de las células enterocromafines de la mucosa gástrica, se manifiestan como
anemia o dolor abdominal. EL tipo 1 y mas frecuente en mujeres se asocia a hipergastrinemia y gastritis atrófica.
6) Carcinoides del colon: La mayoría en COLON DERECHO, siendo el CIEGO el sitio mas afectado. Pueden producir dolor,
anorexia y perdida de peso.
• Como diagnosticas los tumores carcinoides?
1) Cuadro clínico
2) Concentración de serotonina o de sus metabolitos en orina
3) Imagen
4) Se confirma por biopsia
5) Medición del ACIDO 5-HIDROXINDOLACETICO en una muestra de ORINA de 24 hrs, el cual si es
mayor de 10 mg indica EXCESO DE SEROTONINA.
6) En SANGRE se miden los niveles de CROMOGRANINA A, glucoproteina producida por las células derivadas de la
cresta neural y secretada en los tumores neuroendocrinos, relacionada directamente con el volumen tumoral
TUMOR CARCINOIDE
• Cual es el manejo del tumor carcinoide?
1) Depende de la localización del tumor y la presencia de
metástasis
2) Sin metástasis la cirugía es curativa
3) Cuando hay metástasis por lo general es tratamiento
paliativo
4) Quimioterapia a base de cisplatino y etoposido
• Cual es el pronostico del tumor carcinoide?
• R = Depende de la localización, tamaño y metástasis. El
mejor pronóstico lo tienen los carcinoides bronquiales y
apendiculares
SINDROME CARCINOIDE
• Cuando ocurre el síndrome carcinoide?
1) Únicamente cuando hay metástasis a hígado.
2) 10% de pacientes con tumores carcinoides
• Cual es la sintomatología referida mas común de los pacientes con un tumor carcinoide que provoca un síndrome carcinoide?
1) DOLOR ABD INTERMITENTE 70%
2) DIARREA 50%: Es liquida y explosiva.
3) RUBEFACCIÓN 30%: Se caracteriza por eritema intenso o violáceo que se localiza en cara y cuello, se acompaña de sensación
de calor, prurito, lagrimeo, rinorrea o edema facial, se desencadena por estrés, ejercicio, alimentos que contienen tiramina
como quesos y chocolates o por la ingestión de alcohol sobre todo vino tinto.
• Cual es la sustancia vasoactiva mas producida en el síndrome carcinoide?
• R = La SEROTONINA sintetizada a través del triptofano, misma que es metabolizada en hígado y pulmones causando
sintomatología por metastasis.
• Que prueba inicial se recomienda en pacientes con sospecha de síndrome carcinoide?
• R = La medición del acido 5-hidroxindolacetico (5-HIAA, metabolito de la serotonina secretado en orina) y cromogranina A
(encontrada en sangre).
• Cual es el manejo del síndrome carcinoide?
1) Evitar los alimentos que contienen tiramina y la ingestión de alcohol.
2) Si continua con los síntomas, los ANTAGONISTAS DE LOS RECEPTORES DE SEROTONINA (metisergida, ciproheptadina,
ketanserina, ondasetron) o los ANALOGOS DE LA SOMATOSTATINA (octreotide) son los FARMACOS DE ELECCION.
3) La crisis carcinoide puede ser desencadenada por anestesia o manipulación del tumor, produciendo los mismos síntomas que
el síndrome carcinoide, para evitar su aparición, antes de una intervención quirúrgica se administra octreotide de acción
corta en infusión continua 12 hrs antes, durante y 48 hrs después del procedimiento.
TRASTORNOS DE LA PUBERTAD
• Cual es el primer signo físico que aparece de pubertad normal?
1) Mujeres: Botón mamario entre los 8 – 14ª
2) Hombres: Crecimiento testicular entre los 9 – 14ª
• Como se define la pubertad precoz?
• R = Es la aparición de caracteres sexuales secundarios ANTES DE LOS 8ª en las niñas y ANTES
DE LOS 9ª en los niños.
• Como realizas el diagnostico de pubertad precoz?
1) HC
2) Edad ósea, debido a que en la pubertad precoz verdadera esta se encuentra 2 desviaciones
estándar por arriba para su edad y genero.
3) Laboratorios de acuerdo a sospecha clínica
4) Prueba dinámica con administración de GnRH la cual en pacientes con pubertad precoz
verdadera, esperaríamos una respuesta elevada de la hormona LH.
5) El manejo deberá ir encaminado a la causa de origen
• Como se define la pubertad retardada?
• R = Ausencia de desarrollo de caracteres sexuales en una NINA > 13ª y en VARON > 14ª
TRASTORNOS DE LA PUBERTAD
• Cual es la causa mas frecuente de pubertad retardada?
• R = RETRASO CONSTITUCIONAL, los niños que lo presentan generalmente
tienen el antecedente de que alguno de sus padres también presento
retraso en el inicio de la pubertad.
• Como diagnosticas la pubertad retardada?
1) HC
2) Laboratorio de acuerdo a sospecha clinica
• Cual es el manejo de la pubertad retardada?
1) En el retardo constitucional se vigila el momento de presentación de los
caracteres secundarios, si la presión social es importante se podrán utilizar
ENANTATO, CIPIONATO O PROPIONATO DE TETOSTERONA por un periodo
de 3 – 6m .
2) En el caso de hipogonadismo se dará la hormona que haga falta.
HIPERPLASIA SUPRARRENAL
CONGÉNITA
• A que se debe el 90 % de hiperplasia suprarrenal
congénita?
• R = Se debe a la imposibilidad de convertir la
progesterona en 11-desoxicorticosterona mediante la
21-hidroxilasa
• El déficit de la enzima 21-hidroxilasa que ocasiona
fisiopatológicamente?
• R = Desde la conversión de la pregenolona a
aldosterona y cortisol, causando perdida de sal por una
parte y por otra hiperplasia glandular por incremento
del estimulo de la ACTH causando virilización.
HIPERPLASIA SUPRARRENAL CONGÉNITA
• Cual es la etiología del hiperandrogenismo suprarrenal?
• R = Déficit de 21-hidroxilasa, el gen responsable se denomina CYP21B localizado en el brazo
corto del cromosoma 6
• Cual es el cuadro clínico del hiperandrogenismo renal por déficit de 21-hidroxilasa?
A. FORMA CLÁSICA (PERDEDORA DE SAL O VIRILIZANTE SIMPLE): Implica la existencia de
hiperandrogenismo in útero que condiciona la aparición de macrogenitosomia en el varón
y un grado variable de virilización en la mujer. En la forma perdedora de sal existe un
DÉFICIT TANTO DE CORTISOL COMO DE ALDOSTERONA y se manifiesta en ambos sexos por
perdida salina en la etapa neonatal. En la virilizante simple no esta afectada la síntesis de
aldosterona por lo que se mantiene la homeostasis del sodio.
B. FORMA NO CLÁSICA: Existe HIPERANDROGENISMO DE APARICIÓN POSNATAL. Los
síntomas mas frecuentes durante la infancia son pubarquia prematura, piel grasa con acné,
aceleración del crecimiento y maduración ósea y en las niñas aparece hipertrofia de
clítoris, en la adolescencia y la edad adulta las mujeres pueden presentar trastornos
menstruales, hirsutismo, acné, calvicie, ovarios poliquísticos e infertilidad, los varones
afectados pueden cursar con oligoespermia, acné e infertilidad.
• Como diagnosticas el déficit de la enzima 21-hidroxilasa?
• R = Demostración de VALORES ELEVADOS DE 17-HIDROXIPROGESTERONA
• Cual es el manejo del déficit de enzima 21-hidroxilasa?
• R = GLUCOCORTICOIDES y en niños recién nacidos es FLUOROHIDROCORTISONA y en
adultos es a base de PREDNISONA.
TRASTORNOS DE LOS LIPIDOS
• Cual es el hallazgo mas característico de la
hiperquilomicronemia?
• R = Triglicéridos mayor de 1000
• Cual es el hallazgo mas característico de la
hiperbetalipoproteinemia?
• R = Xantomas subcutáneos de los tendones extensores.
• Cual es el hallazgo mas característico de la
hiperlipoproteinemia tipo III?
• R = Xantomas palmares planos.
HEMOCROMATOSIS
• Que sintomatología te sugiere hemocromatosis?
1) High ferritin, hepatomegaly, skin changes, and
diabetes suggest hemochromatosis.
2) The arthritis is characterized by chondrocalcinosis,
but, unlike idiopathic chondrocalcinosis, the hands
are usually involved first. The arthropathy often
progresses despite phlebotomy. Liver disease is
usually the presenting feature.
3) Skin pigmentation is predominantly by melanin.
4) Heart failure is the most common cardiac problem.
ENFERMEDAD DE PAGET
• A 63-year-old asymptomatic woman is investigated for a HIGH
ALKALINE PHOSPHATASE (ALP) level.
• X-rays of the pelvis show multiple porotic and sclerotic lesions with
characteristic whorls of trabeculation. Her excretion of urinary
hydroxyproline is also elevated. Which of the following is the most
likely diagnosis?
• R = Enfermedad de Paget del Hueso
• The elevated ALP AND HYDROXYPROLINE are diagnostic for Paget’s
disease. The bony lesions are blastic and the sacrum and pelvis are
most frequently involved, followed closely by the tibia and femur.
Hypercalcemia can complícate immobilization. The etiology is
unknown, but a viral agent has been postulated. Symptoms may be
absent or severe (pain, deformity). In metastatic cancers of most
types the lesion are lytic, and the other metabolic abnormalities do
not have an elevation in hydroxyproline.
Insulina e Hipoglucemiantes orales.
Hipoglucemiantes orales.
• Sulfonilureas:
– 1a generación: tolbutamida (TBT), clopropramida, tolazamida,
acetohexamida.
– 2a generación: gliburida, glipizida, glibenclamida, glicazida. Son más
potentes.
Insulina e Hipoglucemiantes orales.
Hipoglucemiantes orales.
• Sulfonilureas.
– Mecanismos:
• Estimulan liberación de insulina, disminuyen eliminación hepática,
aumentan somatostatina (disminuye glucagon).
• Regulación a la baja de receptores: al susp pueden tener nuevam
respuesta.
• Bloq canal K sensible a ATP: despolarización: aumenta Ca.
Insulina e Hipoglucemiantes orales.
Hipoglucemiantes orales.
• Sulfonilureas.
– Farmacodinamia.
• VO mejor 30´preprandial.
• Unión PP 90-99% (alb).
• T 1/2 tolazamida 4-7hrs, clorpropramida 24-48hrs. 2da generación 1.5-
5hrs (efecto disminuye glucosa x 12-24hrs).
• Metab hígado. Excreción renal. Cuidado en insuf hepática y renal.
Insulina e Hipoglucemiantes orales.
Hipoglucemiantes orales.
• Sulfonilureas.
– Efectos colaterales.
• 1a generación:40%. Menos frec 2da generación.
• Hipoglucemia.
• Medicamentos: desplazan de PP: sulfonamidas, clofibrato, dicumarol,
salicilatos, fenilbutazona. El alcohol:aumenta su acción..
• Náusea, vómito, ictericia colestásica, agranulocitosis, A aplásica, A
hemolítica, alergia.
• Clorpropramida: dosulfiram 10-15%, hiponatremia 5% x aumentar ADH en
túbulo colector.
• TBT: aumenta 2v riesgo de muerte cardiovascular.
Insulina e Hipoglucemiantes orales.
Hipoglucemiantes orales.
• Sulfonilureas.
– Usos.
• DMIND.
• Contraindicada en DMID, embarazo, lactancia, insuf renal y hepática
significativa.
Insulina e Hipoglucemiantes orales.
Hipoglucemiantes orales.
• Biguanidas.
– Metformin y fenformin.
– Fenformin asociado a ác láctica.
– Metformin T1/2 1.3-4.5 hrs. Máx 3g/d. No produce hipoglucemia, no
libera insulina.
– Aumenta acción perif insulina, inhibe gluconeogénesis, disminuye
absorción gluc intestinal.
– VO.
– No en insuf renal o hepática: ác láctica, hipoxemia cr, insuf cardiaca.
– Efectos colat: 20%. Diarrea, náusea, malestar abd, sabor metálico,
anorexia, disminuye absorción vit B12 y folatos.
Insulina e Hipoglucemiantes orales.
Hipoglucemiantes orales.
• Otros.
– Ciglitazona, pioglitazona, troglitazona: thiazolidinedionas. No dan
hipoglucemia. Disminuyen resistencia a insulina x aumentar
transportadores glucosa.
– Inhib alfa glucosidasa: acarbosa. La inhiben en el borde en cepillo:
inhiben absoción CHOS.
– Efectos colaterales: malaabsorción, flatulencia, distensión abd.
SINDROME HEPATORENAL
• A 44-year-old man presents with worsening abdominal
distension, edema, and jaundice. He has chronic viral
hepatitis B and cirrhosis. Recently he has noticed decreased
urine output despite adequate fluid intake. On examination
he is icteric, the blood pressure is 110/70 mm Hg, pulse
74/min, JVP is 4 cm, heart sounds are normal, and there
are tense ascites and pedal edema. Which of the following
is an early manifestation of hepatorenal syndrome?
• R = Retención de sodio por síndrome hepatorenal
• Hepatorenal syndrome frequently complicates hepatic
failure. Although it can develop gradually, acute renal
failure can also be precipitated by hemodynamic stresses
(bleeding, diuresis). The earliest manifestations are
intrarenal vasoconstriction and avid sodium retention.
EARLY DUMPING
• A 64-year-old man is placed on a low simple sugar diet. He
is experiencing symptoms of crampy abdominal
discomfort, nausea, diarrhea, and diaphoresis 15–30
minutes after eating. Post gastrectomía.
• R = After gastrectomy, avoiding simple sugars and limiting
liquids can ameliorate symptoms of dumping. Early
dumping occurs within 30 minutes of eating and is
characterized by vasomotor symptoms such as palpitations,
tachycardia, lightheadedness, and diaphoresis. Late
dumping includes similar symptoms plus dizziness,
confusion, and even syncope. It occurs 1.2–3 hours after
eating.
DUNNIGAN-KOBBERLING
• Que caracteriza al síndrome de Dunnigan-Kobberling tipo I?
• R = Lipodistrofia parcial familiar AD, durante la pubertad los
pacientes presenta atrofia de las extremidades con cara obesa.
• Que caracteriza al síndrome de Dunnigan-Kobberling tipo II?
• R = Atrofia de toda la grasa subcutánea o ausencia a excepción
de la cara, la cual es muy obesa, pudiendo aparecer hipertrofia
muscular.
• Los pacientes afectados del síndrome de Dunnigan-Kobberling,
a que enfermedades son propensos?
• R = En especial las mujeres. Resistencia a la insulina, DM,
Hipertrigliceridemia, HDL baja y aterosclerosis.
GAUCHER’S DISEASE
• A 26-year-old man is evaluated for back pain and fatigue. He was previously
healthy. On examination, he is pale, there is lumber spine tenderness, and an
enlarged liver (18 cm). He is pancytopenic, and there is a vertebral fracture on
lumbar x-rays. A bone marrow biopsy reveals infiltration with lipid-laden
macrophages (Gaucher cells). Which of the following is the most likely diagnosis?
• Inherited Gaucher’s disease
• The diagnosis is inherited Gaucher’s disease since it is an AUTOSOMAL RECESSIVE
disorder.
• There is no acquired form of the disease. The glucocerebrosides are derived from
lipid catabolites, from the membranes of senescent leukocytes and erythrocytes.
Although the juvenile form may have severe neurologic symptoms (mental
retardation, spasticity, ataxia), the adult form usually has no neurologic symptoms.
Like Tay-Sachs, it is a lysosomal storage disease with a predilection for Ashkenazi
Jews.
OTRAS
• Cual es la forma de presentación mas común de enfermedad crónica renal
relacionada a gota?
• R = Disminución de la habilidad de concentración de la orina
• Que tratamiento podría ayudar a una persona que padece porfiria eritropoyetica?
• R = Betacaroteno aumenta la tolerancia a la luz solar.
• Cuales son los síntomas de la insuficiencia adrenocortical?
• R = Pérdida de peso, fatiga, debilidad nausea y vomito.
• Que sintomatología ocasiona la acumulación de gangliosido y a que anormalidad
metabolica esta relacionada?
• Glycogen storage is not characteristic of Tay- Sachs disease. Ganglioside
accumulation can now be diagnosed by decreased hexosaminidase in peripheral
leukocytes.
• Tay-Sachs is characterized as a lysosomal storage disease. Mental retardation,
seizures, blindness, and a retinal cherry red spot are characteristic. It is most
common in Ashkenazi Jews and is inherited in an autosomal recessive manner.
OTRAS
• Que entidades patológicas causan hiperpigmentación cutánea?
• R = Secresion excesiva de ACTH en Addison, después de una
suprarrenalectomia bilateral en la enfermedad de Cushing ¨Sindrome de
Nelson¨.
• Que fármacos pueden disminuir la ginecomastia?
• R = Inhibidores de la aromatasa como cetrozol, anastrosol y exementano.
• La galactorrea de cualquier etiología a que medicamentos responde?
• R = Bromocriptina o cabergolina
• Que medicamentos se administran al no encontrar testículos a la
palpación?
• R = HCG la cual aumenta la tetosterona
OTRAS
• Que causa y que caracteriza la enfermedad de Mc Ardle?
• R = Es causada por la DISMINUCIÓN DE FOSFORILASA DE LOS MÚSCULOS. Causa
debilidad, calambres musculares, concentraciones elevadas de CK. La
administración de B6 disminuye síntomas.
• Que causa deficiencia de ACTH a nivel supra y extrarenal?
• R = Disminución de la secreción suprarrenal de cortisol, tetosterona y adrenalina.
• Que caracteriza al síndrome de Kallman?
• R = Causa mas común de deficiencia aislada de gonadotropina, déficit recesivo
ligado al X, asociada a Hiposmia por hipoplasia de bulbos olfatorios, agenesia renal
unilateral en 50%. Se trata con análogos de la GNRH como leuprolide
• Cual es el estándar de oro en la disminución de la Hormona de crecimiento?
• R = Hipoglucemia causada por administración de insulina
•
• Que complicación mas frecuente se presenta en la extirpación transfontanelar de
los tumores hipofisiarios?
• R = Hiponatremia
Consejos
1. Leer diariamente y en bloques
2. Adiós Partys un tiempo
3. Has ejercicio y come bien durante el estudio
4. Toma algún curso bueno si tienes la posibilidad
5. Ten Fe.
BIBLIOGRAFIA
• EXARMED
• PAPADAKIS
• CTO
• HARRISON
• AMIR
• USMLE STEPS

Mais conteúdo relacionado

Mais procurados

Insuficiencia Cardiaca Aguda - Revisión y Actualización
Insuficiencia Cardiaca Aguda - Revisión y ActualizaciónInsuficiencia Cardiaca Aguda - Revisión y Actualización
Insuficiencia Cardiaca Aguda - Revisión y ActualizaciónCardioTeca
 
Seminario vasculitis 2015 -Síndromes de vasculitis de vasos pequeños, mediano...
Seminario vasculitis 2015 -Síndromes de vasculitis de vasos pequeños, mediano...Seminario vasculitis 2015 -Síndromes de vasculitis de vasos pequeños, mediano...
Seminario vasculitis 2015 -Síndromes de vasculitis de vasos pequeños, mediano...Juan Carlos Ivancevich
 
(2019-12-12) CETOACIDOSIS DIABETICA Y SINDROME HIPERGLUCEMICO HIPEROSMOLAR (PPT)
(2019-12-12) CETOACIDOSIS DIABETICA Y SINDROME HIPERGLUCEMICO HIPEROSMOLAR (PPT)(2019-12-12) CETOACIDOSIS DIABETICA Y SINDROME HIPERGLUCEMICO HIPEROSMOLAR (PPT)
(2019-12-12) CETOACIDOSIS DIABETICA Y SINDROME HIPERGLUCEMICO HIPEROSMOLAR (PPT)UDMAFyC SECTOR ZARAGOZA II
 
Nefropatia diabetica
Nefropatia diabeticaNefropatia diabetica
Nefropatia diabeticaNancy Barrera
 
Nefropatía Diabética
Nefropatía DiabéticaNefropatía Diabética
Nefropatía DiabéticaJaime Cruz
 
Estado hiperglucemia hiperosmolar
Estado hiperglucemia hiperosmolarEstado hiperglucemia hiperosmolar
Estado hiperglucemia hiperosmolarKevinNava15
 
Acromegalia
AcromegaliaAcromegalia
AcromegaliaUABC
 
35. hipertiroidismo
35. hipertiroidismo35. hipertiroidismo
35. hipertiroidismoxelaleph
 
8. enfermedad de addison
8. enfermedad de addison8. enfermedad de addison
8. enfermedad de addisonMalo Domínguez
 

Mais procurados (20)

Tormenta tiroidea
Tormenta tiroideaTormenta tiroidea
Tormenta tiroidea
 
Miastenia gravis
Miastenia gravisMiastenia gravis
Miastenia gravis
 
Insuficiencia Cardiaca Aguda - Revisión y Actualización
Insuficiencia Cardiaca Aguda - Revisión y ActualizaciónInsuficiencia Cardiaca Aguda - Revisión y Actualización
Insuficiencia Cardiaca Aguda - Revisión y Actualización
 
Seminario vasculitis 2015 -Síndromes de vasculitis de vasos pequeños, mediano...
Seminario vasculitis 2015 -Síndromes de vasculitis de vasos pequeños, mediano...Seminario vasculitis 2015 -Síndromes de vasculitis de vasos pequeños, mediano...
Seminario vasculitis 2015 -Síndromes de vasculitis de vasos pequeños, mediano...
 
(2019-12-12) CETOACIDOSIS DIABETICA Y SINDROME HIPERGLUCEMICO HIPEROSMOLAR (PPT)
(2019-12-12) CETOACIDOSIS DIABETICA Y SINDROME HIPERGLUCEMICO HIPEROSMOLAR (PPT)(2019-12-12) CETOACIDOSIS DIABETICA Y SINDROME HIPERGLUCEMICO HIPEROSMOLAR (PPT)
(2019-12-12) CETOACIDOSIS DIABETICA Y SINDROME HIPERGLUCEMICO HIPEROSMOLAR (PPT)
 
Dermatomiositis polimiositis vasculitis
Dermatomiositis polimiositis vasculitisDermatomiositis polimiositis vasculitis
Dermatomiositis polimiositis vasculitis
 
Nefropatia diabetica
Nefropatia diabeticaNefropatia diabetica
Nefropatia diabetica
 
Nefropatía Diabética
Nefropatía DiabéticaNefropatía Diabética
Nefropatía Diabética
 
Hipertension Portal
Hipertension Portal Hipertension Portal
Hipertension Portal
 
Hipopituitarismo
Hipopituitarismo Hipopituitarismo
Hipopituitarismo
 
Vasculitis 2019 - Sesión Académica del CRAIC
Vasculitis 2019 - Sesión Académica del CRAICVasculitis 2019 - Sesión Académica del CRAIC
Vasculitis 2019 - Sesión Académica del CRAIC
 
crisis tirotoxica tormenta tiroidea
crisis tirotoxica tormenta tiroideacrisis tirotoxica tormenta tiroidea
crisis tirotoxica tormenta tiroidea
 
Estado hiperglucemia hiperosmolar
Estado hiperglucemia hiperosmolarEstado hiperglucemia hiperosmolar
Estado hiperglucemia hiperosmolar
 
Osteoartritis
OsteoartritisOsteoartritis
Osteoartritis
 
(2016 04-05)hipertiroidismo(ppt)
(2016 04-05)hipertiroidismo(ppt)(2016 04-05)hipertiroidismo(ppt)
(2016 04-05)hipertiroidismo(ppt)
 
Hipogonadismo Femenino
Hipogonadismo FemeninoHipogonadismo Femenino
Hipogonadismo Femenino
 
Acromegalia
AcromegaliaAcromegalia
Acromegalia
 
Hipotiroidismo
Hipotiroidismo Hipotiroidismo
Hipotiroidismo
 
35. hipertiroidismo
35. hipertiroidismo35. hipertiroidismo
35. hipertiroidismo
 
8. enfermedad de addison
8. enfermedad de addison8. enfermedad de addison
8. enfermedad de addison
 

Semelhante a Endocrinología: Patología hormonal y tiroides

CLASE DE ENDOCRINO^J SISTEMA NERVIOSO CENTRAL Y OSEO.pptx
CLASE DE ENDOCRINO^J SISTEMA NERVIOSO CENTRAL Y OSEO.pptxCLASE DE ENDOCRINO^J SISTEMA NERVIOSO CENTRAL Y OSEO.pptx
CLASE DE ENDOCRINO^J SISTEMA NERVIOSO CENTRAL Y OSEO.pptxCARMENLOPEZ303
 
Anatomía Patológica del Sistema Endócrino
Anatomía Patológica del Sistema EndócrinoAnatomía Patológica del Sistema Endócrino
Anatomía Patológica del Sistema EndócrinoMZ_ ANV11L
 
patologias_benignas_de_la_tiroide.pptx
patologias_benignas_de_la_tiroide.pptxpatologias_benignas_de_la_tiroide.pptx
patologias_benignas_de_la_tiroide.pptxNombre Apellidos
 
Hipertiroidismo congresonuclear2012 (1)
Hipertiroidismo congresonuclear2012 (1)Hipertiroidismo congresonuclear2012 (1)
Hipertiroidismo congresonuclear2012 (1)Nayeli Laguna
 
Hiperandrogenismo hap-adisson-dislipidemias
Hiperandrogenismo hap-adisson-dislipidemiasHiperandrogenismo hap-adisson-dislipidemias
Hiperandrogenismo hap-adisson-dislipidemiasNancy de la Cruz
 
Sistema endocrino
Sistema endocrinoSistema endocrino
Sistema endocrinoBrianda
 
Caso Clinico endocrinologia trastorno metabolicos.pdf
Caso Clinico  endocrinologia trastorno metabolicos.pdfCaso Clinico  endocrinologia trastorno metabolicos.pdf
Caso Clinico endocrinologia trastorno metabolicos.pdfJonathan Cobos
 
ENFERMEDADES_DE_LA_TIROIDES.pptx .......
ENFERMEDADES_DE_LA_TIROIDES.pptx .......ENFERMEDADES_DE_LA_TIROIDES.pptx .......
ENFERMEDADES_DE_LA_TIROIDES.pptx .......AntonioRivero42
 
CLASE PRACTICA DE PATOLOGIA ENDOCRINA(5).pdf
CLASE PRACTICA DE PATOLOGIA ENDOCRINA(5).pdfCLASE PRACTICA DE PATOLOGIA ENDOCRINA(5).pdf
CLASE PRACTICA DE PATOLOGIA ENDOCRINA(5).pdfCARMENLOPEZ303
 
18-07-12 Actualización
18-07-12 Actualización18-07-12 Actualización
18-07-12 Actualizaciónnachirc
 

Semelhante a Endocrinología: Patología hormonal y tiroides (20)

CLASE DE ENDOCRINO^J SISTEMA NERVIOSO CENTRAL Y OSEO.pptx
CLASE DE ENDOCRINO^J SISTEMA NERVIOSO CENTRAL Y OSEO.pptxCLASE DE ENDOCRINO^J SISTEMA NERVIOSO CENTRAL Y OSEO.pptx
CLASE DE ENDOCRINO^J SISTEMA NERVIOSO CENTRAL Y OSEO.pptx
 
Suprarrenal
SuprarrenalSuprarrenal
Suprarrenal
 
Anatomía Patológica del Sistema Endócrino
Anatomía Patológica del Sistema EndócrinoAnatomía Patológica del Sistema Endócrino
Anatomía Patológica del Sistema Endócrino
 
patologias_benignas_de_la_tiroide.pptx
patologias_benignas_de_la_tiroide.pptxpatologias_benignas_de_la_tiroide.pptx
patologias_benignas_de_la_tiroide.pptx
 
Hipertiroidismo congresonuclear2012 (1)
Hipertiroidismo congresonuclear2012 (1)Hipertiroidismo congresonuclear2012 (1)
Hipertiroidismo congresonuclear2012 (1)
 
Hipotiroidismo
Hipotiroidismo Hipotiroidismo
Hipotiroidismo
 
Hiperandrogenismo hap-adisson-dislipidemias
Hiperandrogenismo hap-adisson-dislipidemiasHiperandrogenismo hap-adisson-dislipidemias
Hiperandrogenismo hap-adisson-dislipidemias
 
Endocrinologia III
Endocrinologia IIIEndocrinologia III
Endocrinologia III
 
Sistema endocrino
Sistema endocrinoSistema endocrino
Sistema endocrino
 
gomez
gomezgomez
gomez
 
Caso Clinico endocrinologia trastorno metabolicos.pdf
Caso Clinico  endocrinologia trastorno metabolicos.pdfCaso Clinico  endocrinologia trastorno metabolicos.pdf
Caso Clinico endocrinologia trastorno metabolicos.pdf
 
ENFERMEDADES_DE_LA_TIROIDES.pptx .......
ENFERMEDADES_DE_LA_TIROIDES.pptx .......ENFERMEDADES_DE_LA_TIROIDES.pptx .......
ENFERMEDADES_DE_LA_TIROIDES.pptx .......
 
Amenorrea 2
Amenorrea 2Amenorrea 2
Amenorrea 2
 
CLASE PRACTICA DE PATOLOGIA ENDOCRINA(5).pdf
CLASE PRACTICA DE PATOLOGIA ENDOCRINA(5).pdfCLASE PRACTICA DE PATOLOGIA ENDOCRINA(5).pdf
CLASE PRACTICA DE PATOLOGIA ENDOCRINA(5).pdf
 
Tiroides (juan carlos)
Tiroides (juan carlos)Tiroides (juan carlos)
Tiroides (juan carlos)
 
Tiroides
Tiroides Tiroides
Tiroides
 
18-07-12 Actualización
18-07-12 Actualización18-07-12 Actualización
18-07-12 Actualización
 
Hipopituitarismo Act 1
Hipopituitarismo Act 1Hipopituitarismo Act 1
Hipopituitarismo Act 1
 
Clase 6-signed
Clase 6-signedClase 6-signed
Clase 6-signed
 
Hipotiroidismo
HipotiroidismoHipotiroidismo
Hipotiroidismo
 

Mais de Alejandro Sandoval (13)

Hipertension en el embarazo ACOG 2013
Hipertension en el embarazo ACOG 2013Hipertension en el embarazo ACOG 2013
Hipertension en el embarazo ACOG 2013
 
ENARM Obstetricia ginecologia
ENARM Obstetricia ginecologiaENARM Obstetricia ginecologia
ENARM Obstetricia ginecologia
 
PEDIATRIA ENARM
PEDIATRIA ENARMPEDIATRIA ENARM
PEDIATRIA ENARM
 
10.NEUMOLOGIA
10.NEUMOLOGIA10.NEUMOLOGIA
10.NEUMOLOGIA
 
8.NEUROLOGIA
8.NEUROLOGIA8.NEUROLOGIA
8.NEUROLOGIA
 
7.DERMATOLOGIA
7.DERMATOLOGIA7.DERMATOLOGIA
7.DERMATOLOGIA
 
6.INFECTOLOGIA
6.INFECTOLOGIA6.INFECTOLOGIA
6.INFECTOLOGIA
 
5.NEFROLOGIA
5.NEFROLOGIA5.NEFROLOGIA
5.NEFROLOGIA
 
4.GASTROENTEROLOGIA Y CIRUGIA
4.GASTROENTEROLOGIA Y CIRUGIA4.GASTROENTEROLOGIA Y CIRUGIA
4.GASTROENTEROLOGIA Y CIRUGIA
 
3.REUMATOLOGIA
3.REUMATOLOGIA3.REUMATOLOGIA
3.REUMATOLOGIA
 
2.CARDIOLOGIA
2.CARDIOLOGIA2.CARDIOLOGIA
2.CARDIOLOGIA
 
1.HEMATOLOGIA
1.HEMATOLOGIA1.HEMATOLOGIA
1.HEMATOLOGIA
 
ENARM COMPENDIO
ENARM  COMPENDIOENARM  COMPENDIO
ENARM COMPENDIO
 

Último

fisiologia aparato digestivo-MEDICINA.....
fisiologia aparato digestivo-MEDICINA.....fisiologia aparato digestivo-MEDICINA.....
fisiologia aparato digestivo-MEDICINA.....kelyacerovaldez
 
INFECCION DE TRACTO URINARIO (ITU) EN GESTANTES
INFECCION DE TRACTO URINARIO (ITU) EN GESTANTESINFECCION DE TRACTO URINARIO (ITU) EN GESTANTES
INFECCION DE TRACTO URINARIO (ITU) EN GESTANTESangelojosue
 
ABDOMEN AGUDO Quirúrgico, etiologia.pptx
ABDOMEN AGUDO Quirúrgico, etiologia.pptxABDOMEN AGUDO Quirúrgico, etiologia.pptx
ABDOMEN AGUDO Quirúrgico, etiologia.pptxNikolaiChoqueAlarcn
 
SEMANA 1 GENERALIDADES Y TERMINOLOGIAS EN BIOSEGURIDAD.pptx
SEMANA 1 GENERALIDADES Y TERMINOLOGIAS EN BIOSEGURIDAD.pptxSEMANA 1 GENERALIDADES Y TERMINOLOGIAS EN BIOSEGURIDAD.pptx
SEMANA 1 GENERALIDADES Y TERMINOLOGIAS EN BIOSEGURIDAD.pptxanny545237
 
CLASIFICACION DEL RECIEN NACIDO NIÑO.pptx
CLASIFICACION DEL RECIEN NACIDO NIÑO.pptxCLASIFICACION DEL RECIEN NACIDO NIÑO.pptx
CLASIFICACION DEL RECIEN NACIDO NIÑO.pptxMairimCampos1
 
Pelvis y periné, estructura osea, musculos
Pelvis y periné, estructura osea, musculosPelvis y periné, estructura osea, musculos
Pelvis y periné, estructura osea, musculosElkinJavierSalcedoCo
 
Regulación emocional. Salud mental. Presentaciones en la red. Slideshare. Ens...
Regulación emocional. Salud mental. Presentaciones en la red. Slideshare. Ens...Regulación emocional. Salud mental. Presentaciones en la red. Slideshare. Ens...
Regulación emocional. Salud mental. Presentaciones en la red. Slideshare. Ens...Lorena Avalos M
 
Bartonelosis-Medicina tropical-Medicina.pptx
Bartonelosis-Medicina tropical-Medicina.pptxBartonelosis-Medicina tropical-Medicina.pptx
Bartonelosis-Medicina tropical-Medicina.pptx Estefa RM9
 
Sala Situacional Nacional - MINSA Perú 2024
Sala Situacional Nacional - MINSA Perú 2024Sala Situacional Nacional - MINSA Perú 2024
Sala Situacional Nacional - MINSA Perú 2024Miguel Yan Garcia
 
MTC Reinos mutante MADERA FUEGO TIERRA.pdf
MTC Reinos mutante MADERA FUEGO TIERRA.pdfMTC Reinos mutante MADERA FUEGO TIERRA.pdf
MTC Reinos mutante MADERA FUEGO TIERRA.pdfMelindaSayuri
 
Clase 11 Articulaciones de la Cabeza 2024.pdf
Clase 11 Articulaciones de la Cabeza 2024.pdfClase 11 Articulaciones de la Cabeza 2024.pdf
Clase 11 Articulaciones de la Cabeza 2024.pdfgarrotamara01
 
CLASE 1 MASAJE DESCONTRACTURANTE2016.pdf
CLASE 1 MASAJE DESCONTRACTURANTE2016.pdfCLASE 1 MASAJE DESCONTRACTURANTE2016.pdf
CLASE 1 MASAJE DESCONTRACTURANTE2016.pdfdanicanelomasoterapi
 
EJERCICIOS DE BUERGUER ALLEN FISIOTERAPIApptx
EJERCICIOS DE BUERGUER ALLEN FISIOTERAPIApptxEJERCICIOS DE BUERGUER ALLEN FISIOTERAPIApptx
EJERCICIOS DE BUERGUER ALLEN FISIOTERAPIApptxMaria969948
 
Tejido muscular- Histologia-Geneser.pptx
Tejido muscular- Histologia-Geneser.pptxTejido muscular- Histologia-Geneser.pptx
Tejido muscular- Histologia-Geneser.pptx Estefa RM9
 
Triptico sobre que son y como PrevencionITS.pptx
Triptico sobre que son y como PrevencionITS.pptxTriptico sobre que son y como PrevencionITS.pptx
Triptico sobre que son y como PrevencionITS.pptxLysMedina
 
Perfil-Sensorial-2-Child. Versión actualizada
Perfil-Sensorial-2-Child.  Versión actualizadaPerfil-Sensorial-2-Child.  Versión actualizada
Perfil-Sensorial-2-Child. Versión actualizadaNadiaMocio
 
Analisis Evolución Dengue - MINSA Perú 2024
Analisis Evolución Dengue - MINSA Perú 2024Analisis Evolución Dengue - MINSA Perú 2024
Analisis Evolución Dengue - MINSA Perú 2024Miguel Yan Garcia
 
Acceso venoso periferico, caracteristicas y funciones
Acceso venoso periferico, caracteristicas y funcionesAcceso venoso periferico, caracteristicas y funciones
Acceso venoso periferico, caracteristicas y funcionesDamaryHernandez5
 
A- LIBRO DE GINECOLOGIA Y OBSTRETICIA DE SEGO (2).pdf
A- LIBRO DE GINECOLOGIA Y OBSTRETICIA DE SEGO (2).pdfA- LIBRO DE GINECOLOGIA Y OBSTRETICIA DE SEGO (2).pdf
A- LIBRO DE GINECOLOGIA Y OBSTRETICIA DE SEGO (2).pdfcoloncopias5
 

Último (20)

fisiologia aparato digestivo-MEDICINA.....
fisiologia aparato digestivo-MEDICINA.....fisiologia aparato digestivo-MEDICINA.....
fisiologia aparato digestivo-MEDICINA.....
 
INFECCION DE TRACTO URINARIO (ITU) EN GESTANTES
INFECCION DE TRACTO URINARIO (ITU) EN GESTANTESINFECCION DE TRACTO URINARIO (ITU) EN GESTANTES
INFECCION DE TRACTO URINARIO (ITU) EN GESTANTES
 
ABDOMEN AGUDO Quirúrgico, etiologia.pptx
ABDOMEN AGUDO Quirúrgico, etiologia.pptxABDOMEN AGUDO Quirúrgico, etiologia.pptx
ABDOMEN AGUDO Quirúrgico, etiologia.pptx
 
SEMANA 1 GENERALIDADES Y TERMINOLOGIAS EN BIOSEGURIDAD.pptx
SEMANA 1 GENERALIDADES Y TERMINOLOGIAS EN BIOSEGURIDAD.pptxSEMANA 1 GENERALIDADES Y TERMINOLOGIAS EN BIOSEGURIDAD.pptx
SEMANA 1 GENERALIDADES Y TERMINOLOGIAS EN BIOSEGURIDAD.pptx
 
CLASIFICACION DEL RECIEN NACIDO NIÑO.pptx
CLASIFICACION DEL RECIEN NACIDO NIÑO.pptxCLASIFICACION DEL RECIEN NACIDO NIÑO.pptx
CLASIFICACION DEL RECIEN NACIDO NIÑO.pptx
 
Pelvis y periné, estructura osea, musculos
Pelvis y periné, estructura osea, musculosPelvis y periné, estructura osea, musculos
Pelvis y periné, estructura osea, musculos
 
Regulación emocional. Salud mental. Presentaciones en la red. Slideshare. Ens...
Regulación emocional. Salud mental. Presentaciones en la red. Slideshare. Ens...Regulación emocional. Salud mental. Presentaciones en la red. Slideshare. Ens...
Regulación emocional. Salud mental. Presentaciones en la red. Slideshare. Ens...
 
Neumonia complicada en niños y pediatria vrs neumonia grave, gérmenes, nuevas...
Neumonia complicada en niños y pediatria vrs neumonia grave, gérmenes, nuevas...Neumonia complicada en niños y pediatria vrs neumonia grave, gérmenes, nuevas...
Neumonia complicada en niños y pediatria vrs neumonia grave, gérmenes, nuevas...
 
Bartonelosis-Medicina tropical-Medicina.pptx
Bartonelosis-Medicina tropical-Medicina.pptxBartonelosis-Medicina tropical-Medicina.pptx
Bartonelosis-Medicina tropical-Medicina.pptx
 
Sala Situacional Nacional - MINSA Perú 2024
Sala Situacional Nacional - MINSA Perú 2024Sala Situacional Nacional - MINSA Perú 2024
Sala Situacional Nacional - MINSA Perú 2024
 
MTC Reinos mutante MADERA FUEGO TIERRA.pdf
MTC Reinos mutante MADERA FUEGO TIERRA.pdfMTC Reinos mutante MADERA FUEGO TIERRA.pdf
MTC Reinos mutante MADERA FUEGO TIERRA.pdf
 
Clase 11 Articulaciones de la Cabeza 2024.pdf
Clase 11 Articulaciones de la Cabeza 2024.pdfClase 11 Articulaciones de la Cabeza 2024.pdf
Clase 11 Articulaciones de la Cabeza 2024.pdf
 
CLASE 1 MASAJE DESCONTRACTURANTE2016.pdf
CLASE 1 MASAJE DESCONTRACTURANTE2016.pdfCLASE 1 MASAJE DESCONTRACTURANTE2016.pdf
CLASE 1 MASAJE DESCONTRACTURANTE2016.pdf
 
EJERCICIOS DE BUERGUER ALLEN FISIOTERAPIApptx
EJERCICIOS DE BUERGUER ALLEN FISIOTERAPIApptxEJERCICIOS DE BUERGUER ALLEN FISIOTERAPIApptx
EJERCICIOS DE BUERGUER ALLEN FISIOTERAPIApptx
 
Tejido muscular- Histologia-Geneser.pptx
Tejido muscular- Histologia-Geneser.pptxTejido muscular- Histologia-Geneser.pptx
Tejido muscular- Histologia-Geneser.pptx
 
Triptico sobre que son y como PrevencionITS.pptx
Triptico sobre que son y como PrevencionITS.pptxTriptico sobre que son y como PrevencionITS.pptx
Triptico sobre que son y como PrevencionITS.pptx
 
Perfil-Sensorial-2-Child. Versión actualizada
Perfil-Sensorial-2-Child.  Versión actualizadaPerfil-Sensorial-2-Child.  Versión actualizada
Perfil-Sensorial-2-Child. Versión actualizada
 
Analisis Evolución Dengue - MINSA Perú 2024
Analisis Evolución Dengue - MINSA Perú 2024Analisis Evolución Dengue - MINSA Perú 2024
Analisis Evolución Dengue - MINSA Perú 2024
 
Acceso venoso periferico, caracteristicas y funciones
Acceso venoso periferico, caracteristicas y funcionesAcceso venoso periferico, caracteristicas y funciones
Acceso venoso periferico, caracteristicas y funciones
 
A- LIBRO DE GINECOLOGIA Y OBSTRETICIA DE SEGO (2).pdf
A- LIBRO DE GINECOLOGIA Y OBSTRETICIA DE SEGO (2).pdfA- LIBRO DE GINECOLOGIA Y OBSTRETICIA DE SEGO (2).pdf
A- LIBRO DE GINECOLOGIA Y OBSTRETICIA DE SEGO (2).pdf
 

Endocrinología: Patología hormonal y tiroides

  • 2. PATOLOGIA HORMONAL • Cual es el cuadro clínico del hipopitituarismo? 1) Astenia 2) INTOLERANCIA AL FRIO 3) Hipoglucemia 4) AMENORREA 5) Disminución de la libido 6) Ausencia de bello axilar • Cual es la causa mas común de sobreproducción de hormonas de la hipófisis anterior? • R = Un adenoma hipofisario • Como determinas de acuerdo a la medida de un adenoma hipofisario si es micro o macroadenoma? • R = Mas de 1 cm es macro adenoma, lo contrario es micro • De los adenomas hipofisarios cual es el tipo histológico mas común? • R = 1 ero PRODUCTOR DE PROLACTINA 30 % , 2do Productor de GH • Que manifestaciones clínicas se asocian a la hiperprolactinemia? • R = AMENORREA, GALACTORREA, perdida de la libido, ABORTOS DE REPETICIÓN, ESTERILIDAD, datos de hiperandrogenismo como HIRSUTISMO Y ACNÉ. • Que tipo de fármacos o padecimientos producen hiperprolactnemia? • R = PROLACTINOMA. Antagonistas de los receptores de dopamina como los NEUROLÉPTICOS (fenotiazinas, HALOPERIDOL) y ANTIGUOS ANTIHIPERTENSIVOS COMO LA RESEPRINA que inhiben el almacenamiento del mediador. También los ESTRÓGENOS, la IRA y el HIPOTIROIDISMO.
  • 3. PATOLOGIA HORMONAL • A que se le conoce como síndrome de Nelson? • R = Es el resultado de la perdida del EFECTO INHIBIDOR DE LOS CORTICOSTEROIDES SUPRARRENALES sobre un microadenoma corticotrofo preexistente. • Que ocasiona una apoplejía hipofisiaria y como se manifiesta? • R = Una HEMORRAGIA brusca en la glándula que ocurre a menudo secundario a adenoma. Se presenta de manera brusca con un INTENSO DOLOR DE CABEZA, DIPLOPÍA E HIPOPITUITARISMO. • Que defecto genético en niños se relaciona a la deficiencia de hormonas hipofisarias? • R = Deficiencia en el gen que codifica ¨PIT-1¨ • Cual es la forma de presentación más común hiposecreción de la pituitaria anterior? • R = AMENORREA • Cuales son los receptores hormonales mas importantes a buscar para iniciar el tratamiento de cáncer de mama? • R = Estrogenos y progesterona. • Que puede ocasionar en hombres elevación de la prolactina? • R = Ausencia de erección con disminución del grosor del pene. • Que manifestaciones clínicas da el déficit congénito de leptina? • R = Obesidad, esta se encarga de dar la SENSACIÓN DE LA SACIEDAD. • Como actúa la hormona de alfa melanocito? • R = Es un neuropeptido que REGULA LA INGESTIÓN DE ALIMENTOS
  • 4. HIPERPROLACTINEMIA • Cual es una hormona inhibidora de la prolactina? • R = DOPAMINA, el aumento de estrógenos la inhibe. • Cuales son los signos y síntomas de hiperprolactinemia? 1) Hombres: DISFUNCIÓN ERÉCTIL, HIPOGONADISMO HIPOGONADOTROFICO, INFERTILIDAD y disminución de la libido. En ocasiones GINECOMASTIA SIN GALACTORREA. 2) Mujeres: OLIGOMENORREA, amenorrea, GALACTORREA, si el hipogonadismo no es tratado da osteoporosis. • Que lab se solicitan en hiperprolactinemia y que padecimientos se encuentran relacionados? • R = HCG en caso de embarazo, HIPOTIROIDISMO, IR y cirrosis. • Que estudio realizas en hiperprolactinemia no asociada a hipotiroidismo, embarazo o inducida por fármacos? • R = IRMbuscando micro <2 cm o macroadenoma hipofisiario >3 cm. • Que medicamentos no deben recibir las mujeres con prolactinomas? • R = ESTRÓGENOS (<2cm no afectan), TETOSTERONAo EMBARAZO. • Cual es el tratamiento para hiperprolactinemia? • R = BROMOCRIPTINA. Antagonistas de la dopamina como CENTENO O CABERGOLINA al acostarse, Quinagolida a los intolerantes derivados del cornezuelo de centeno
  • 5.
  • 6. SINDROME METABOLICO • PARA EL DIAGNOSTICO DE ESTE SÍNDROME ES NECESARIO LA PRESENCIA DE 3 O MAS DE LOS SIGUIENTES CRITERIOS: – OBESIDAD ABDOMINAL MUJERES > 88 CMS Y HOMBRE > 102 CMS. – TRIACILGLICERIDOS >O IGUAL A 150 MG/DL O CON TRATAMIENTO. – HDL <40 MG/DL (HOMBRES) Y < 50 MG/DL (MUJERES). – TENSIÓN ARTERIAL SISTÓLICA >O IGUAL A 130 MMHG Y DIASTÓLICA >O IGUAL A 85 MMHG. – GLICEMIA > 110 MG/DL O CON TRATAMIENTO HIPOGLICEMIANTE.
  • 7. SSIADH • En que región del hipotálamo se secreta la ADH y que funciones tiene? 1) En el NÚCLEO SUPRA ÓPTICO 2) La hormona antidiurética (ADH), o arginina vasopresina (AVP), es una hormona LIBERADA PRINCIPALMENTE EN RESPUESTA A CAMBIOS EN LA OSMOLARIDAD SÉRICA O EN EL VOLUMEN SANGUÍNEO. También conocida como argipresina. Hace que los riñones conserven agua mediante la concentración de orina y la reducción de su volumen, estimulando la reabsorción de agua. Recibe su nombre de esta importante función como regulador homeostásico de fluidos. • Cual es la forma mas común de HIPONATREMIA NORMOVOLEMICA? • R = SSIADH • Cual es el cc del SSIADH? 1) Neuromusculares: IRRITABILIDAD, debilidad muscular, letargo, CONFUSIÓN y coma. 2) Gastrointestinales: NAUSEA, VOMITO y anorexia 3) La intensidad de los síntomas se relaciona con el grado de hiponatremia • Como diagnosticas SSIADH? 1) Por exclusión 2) Prueba de sobrecarga hídrica y hallazgos de niveles plasmáticos inapropiadamente elevados de vasopresina en relación a la osmolaridad plasmática • Cual es el manejo del SSIADH? 1) RESTRICCIÓN HÍDRICA 500-1000 ml/24 hrs 2) DEMECLOCICLINA (inhibe la respuesta renal a la vasopresina)
  • 8. PATOLOGIA DE TIROIDES • En que pacientes aumenta el riesgo de que un nódulo tiroideo palpable sea maligno? 1) Antecedentes de radiación de cabeza y cuello, 2) AHF de cáncer tiroideo o asociada a linfadenopatia. • Que patologías te pueden causar hipertiroidismo? • R = Graves, bocio multinodular toxico, nódulos hiperfuncionantes, tiroiditis sub aguda. • Cuales son los signos y síntomas del bocio endémico? • R = Focos subesternales pueden ocasionar: Compresión traqueal, insuficiencia respiratoria, sx de vena cava superior, hemorragia GI + varices esofágicas, parálisis de los nervios frénico, faríngeo recurrente o sx de Horner. • Cuales son los datos de lab de bocio endémico? • R = T4 sérica suele ser normal, TSH normal, Sin embargo varían con hipo/hipertiroidismo
  • 9.
  • 10. PATOLOGIA DE TIROIDES • Que patologias o farmacos pueden ocasionar bocio en hipotiroidismo? • R = Tiroiditis de Hashimoto, deficiencia de yodo, FÁRMACOS BOCIOGENOS (LITIO, yoduro, propiltiuracilo, metimazol, fenilbutazona, sulfonamidas, AMIODARONA) , Tiroiditis sub aguda de Quervain • En que situaciones el bocio se encuentra ausente en hipotiroidismo? • R = Secreción hipofisiaria disminuida de TSH, Destrucción de la glándula por cirugía o radiación. • Cual es el cuadro clínico del hipotiroidismo? 1) Tempranos: fatiga, LETARGO, debilidad, artralgias, mialgias, calambres musculares, INTOLERANCIA AL FRIO, ROTS <. 2) Tardios: dislalia, ESTREÑIMIENTO, edema periférico, palidez, RONQUERA, AUMENTO DE PESO, amenorrea-menorragia, GLOSITIS. 3) DERRAME PLEURAL, PERITONEAL O PERICARDICO. CORAZON DE MIXEDEMA POR DERRAME PERICARDICO. • Cuales son las complicaciones del hipotiroidismo? 1) Cardiacas 2) Locura del mixedema: psicosis orgánica con delirios paranoides, coma mixedematoso desencadenado por infeccioso o traumatismo que se acompaña de hipo/glucemia-natremia. • Cual es el manejo del hipotiroidismo? • R = Levotiroxina
  • 11. PATOLOGIA DE TIROIDES • Que características clínicas acompañan a la tirotoxicosis? • R = Nerviosismo, PALPITACIONES, TAQUICARDIA, fatiga, debilidad, pérdida de peso con buen apetito, DIARREA, INTOLERANCIA AL CALOR, PIEL CALIENTE, transpiración excesiva, labilidad emocional, alteraciones menstruales, TEMBLOR FINO EN LAS MANOS, alteraciones oculares y aumento variable de la glándula. FA. • Como se manifiesta la CRISIS O TORMENTA TIROIDEA? • SE MANIFIESTA POR DELIRIO, TAQUICARDIA, VOMITO, DIARREA y FIEBRE. • Cual es el manejo de la tormenta tiroidea? • R = Dosis altas de PROPILTIUORACILO, yodo o contrastes yodados, PROPANOLOL Y DEXAMETASONA • Cuales son los análisis mas específicos para enfermedad de Graves? • R = TSH-R AB 2da generación y TSH-R recombinante son > 75% • Que diferencia hay entre la captación de yodo de las siguientes patologías? 1) Enfermedad de Graves y bocio multinodular toxico: ALTO 2) Tiroiditis sub aguda: BAJO
  • 12. PATOLOGIA DE TIROIDES • La enfermedad de GRAVES es la causa mas común de TIROTOXICOSIS, que la caracteriza? 1) Enfermedad de Basedow en Europa. 2) ETIOLOGÍA AUTOINMUNE, aumento de hormonas tiroideas, AUMENTO DE GLÁNDULA, AC VS TSH. 3) Mujeres 8:1. SE RELACIONA con enfermedades autoinmunes como ANEMIA PERNICIOSA, MIASTENIA GRAVE. 4) Asociación con HLA-B8 y HLA DR3, AC VS PEROXIDASA, AC VS TIROGLOBULINA y ANA están presentes • Que es la enfermedad de Plummer en tirotoxicosis? • R = Son adenomas tóxicos tiroideos simples • Que es la enfermedad de Jodbasedow en tirotoxicosis? • R = Hipertiroidismo inducido por yodo • Que patologías extraglandulares originan tirotoxicosis? • R = Aumento en la HCG, EMBARAZO MOLAR, CORIOCARCINOMAy neoplasias testiculares. Administración de AMIODARONA en arritmias. • Cuales son los signos y síntomas de tirotoxicosis? 1) Mirada fija, CAÍDA DE PARPADO, taquicardia o FA, TEMBLORES FINOS, PIEL HUMEDA/CALIENTE. 2) GRAVES: Oftalmopatia con quemosis, conjuntivitis y PROPTOSIS. DERMOPATIA “MIXEDEMA PRETIBIAL” con ACUMULACIÓN DE GLUCOSAMINOGLUCANOS que ocasiona edema con textura rugosa.
  • 13. PATOLOGIA DE TIROIDES • Cual es el manejo de la enfermedad de Graves? • 1. PROPANOLOL: taquicardia, temblores y ansiedad • 2. Derivados de la tiourea (METIMAZOL O PROPILTIURACILO): Puede causar AGRANULOSITOSIS • 3. Metimazol: < riesgo de necrosis hepática, fulminante • 4. Propiltiuracilo: Es de ELECCIÓN en EMBARAZO O LACTANCIA • 5. Agente de contraste yodados: ayudan en tirotoxicosis de cualquier etiología los cuales inhibe la trimonoyodinacion de T4 por lo tanto aumenta T3 y disminuye tirotoxicosis • 6. YODO RADIACTIVO: destruye tejido tiroideo sobre activo • Que medidas se usan para complicaciones de enfermedad de Graves: • 1. Oftalmopatia de Graves: EXOFTALMUS; ESTEROIDES, progresivo-radioterapia • 2. Cardiacas: taquicardia sinusal; PROPANOLOL • 3. Insuficiencia cardiaca: DIURÉTICOS Y DIGOXINA • Que causa dermopatia de Graves? • R = Engrosamiento de piel por glucosaminoglucanos, se le conoce como MIXEDEMA PRETIBIAL Y SU TRATAMIENTO SON ESTEROIDES TÓPICOS.
  • 14. PATOLOGIA TIROIDES • Que caracteriza en la tirotoxicosis a la PARÁLISIS PERIÓDICA HIPOKALEMICA TIROTOXICA? • R = PARÁLISIS FLÁCIDA SIMÉTRICA REPENTINA, HIPOKALEMIA E HIPOFOSFATEMIA. El tratamiento es con PROPANOLOL que revierte la parálisis en 3 hrs. Dextrosa IV o CHBTS lo agravan. • Cual es el manejo de los nódulos tiroideos solitarios tóxicos? • R = PROPANOLOL, YODO RADIACTIVO • Cual es el manejo del bocio multinodular toxico? • R = PROPANOLOL, YODO RADIACTIVO • Cual es el cc de la TIROIDITIS DE HASHIMOTO O TIROIDITIS LINFOCITICA CRÓNICA y a que enfermedad progresa? 1) Tiroides con AUMENTO DE TAMAÑO SIN DOLOR, de consistencia dura, ASIMÉTRICO y fijo 2) PROGRESIÓN A HIPOTIROIDISMO. 3) Depresión, fatiga crónica, XEROSTOMÍA, XEROFTALMIA. • Que anticuerpos son los principales implicados en la tiroiditis de Hashimoto? • R = AC ANTIMIELOPEROXIDASA y LINFOCITOS TCD 4
  • 15. PATOLOGIA DE TIROIDES • Cual es el manejo de tiroiditis de Hashimoto al causar hipotiroidismo, bocio y Hashitoxicosis? 1) Hipotiroidismo: LEVOTIROXINA 2) Bocio: T4 para disminuir tamaño de glándula 3) Hashitoxicosis: Aumenta liberación de T4 por acumulo, PROPANOLOL o yodato sódico • Cuales son los medicamentos principalmente vinculados con la tiroiditis inducida por fármacos? • R = AMIODARONA, Interferon alfa, LITIO e interleucina 2. • Cual es el cuadro clínico de la tiroiditis por fármacos? • R = Tiroiditis INDOLORA • Que caracteriza a la TIROIDITIS DE RIEDEL /tiroiditis fibrosa invasiva, tiroiditis leñosa, tiroiditis lingeosa o tiroiditis invasiva? 1) Provoca HIPOTIROIDISMO e HIPOPARATIROIDISMO. 2) Se presenta en EDAD AVANZADA. 3) GLÁNDULA DURA CON ADHERENCIAS al cuello ocasionando DISNEA, DISFAGIA, DOLOR Y RONQUERA. 4) Los trastornos incluyen FIBROSIS RETROPERITONEAL. • Cuales son las características laboratoriales en la producción de T3 o T4 de distinción entre los padecimientos causantes de hipertiroidismo? 1) TIROIDITIS SUB AGUDA (QUERVAIN)/ HASHIMOTO: >T4 + que T3 2) GRAVES/ BOCIO NODULAR TOXICO: >T3
  • 16. PATOLOGIA DE TIROIDES • Cual es la causa de la tiroiditis aguda o supurativa? • R = Infecciosa • Cual es el cuadro clínico de la tiroiditis aguda o supurativa? • R = El paciente cursa con FIEBRE, disfagia, disfonía, DOLOR EN LA REGIÓN ANTERIOR DEL CUELLO, eritema y GLANDULA TIROIDEA SENSIBLE • Que muestra la biopsia en la tiroiditis aguda o supurativa? • R = INFILTRADO LEUCOCITARIO • Cual es el manejo de tiroiditis supurativa? • R = Antibióticos + drenaje quirúrgico • Cual es la etiología de la tiroiditis subaguda posparto? • R = Es causada por la INFLAMACIÓN LINFOCITICA de la tiroides en los primeros 3 A 6M POSTERIORES AL PARTO.
  • 17. PATOLOGIA DE TIROIDES • Cual es el cc de la tiroiditis subaguda posparto? • R = EL BOCIO es INDOLORO • Como se encuentra la VSG en la tiroiditis subaguda posparto? • R = VSG NORMAL que la DIFERENCIA DE LA DE QUERVAIN. • Cual es el manejo de la tiroiditis subaguda posparto? 1) Rara vez requiere tratamiento 2) En la FASE DE TIROTOXICOSIS es útil un BETABLOQUEADOR 3) En la fase de HIPOTIROIDISMO puede usarse LEVOTIROXINA 4) Los fármacos anti tiroideos y los esteroides NO ESTÁN INDICADOS • Cual es la etiología de la tiroiditis subaguda o de Quervain? 1) Suele ser precedida por un PROCESO INFECCIOSO DE LAS VAS 2) Se ha relacionado con ECHOVIRUS, ADENOVIRUS, COXACKIE VIRUS Y VIRUS DE LA PAROTIDITIS 3) La edad de presentación es entre 20-40ª
  • 18. PATOLOGIA DE TIROIDES • Cual es el cuadro clínico de la TIROIDITIS DE QUERVAIN? EL cuadro clínico comienza con pródromos de : 1) MIALGIAS GENERALIZADAS 2) Faringitis 3) Fatigas 4) FIEBRE 5) DOLOR Y EDEMA DEL CUELLO • Como se encuentra la VSG en la tiroiditis de Quervain? 1) ELEVACIÓN DE LA VSG 2) Elevación de PCR • Cual es el manejo de tiroiditis subaguda o de Quervain? • R = ASA es de elección, PROPANOLOL para SÍNTOMAS TIROTOXICOS • Cual es el manejo del ESTROMA DE YEDEL? • R = TAMOXIFENO de elección por años
  • 19. PATOLOGIA DE TIROIDES • Un teratoma ovárico que patología puede ocasionar, relacionado a la producción de hormonas? • R = TIROTOXICOSIS por producción de TSH • Como afecta el aumento de hormonas tiroideas a los huesos? • R = Aumenta la estimulación de la RESORCIÓN ÓSEA. • Cuales son las manifestaciones clínicas del CRETINISMO? • R = Alteración del desarrollo esquelético y del SNC, CARA TOSCA, PROTRUSIÓN DE LA LENGUA Y HERNIA UMBILICAL. • A que se le llama síndrome de plumer? • R = Cuando uno de los nódulos del BOCIO MULTINODULAR se hace HIPERFUNCIONANTE (bocio multinodular toxico), puesto que la mayoría de los pacientes son eutiroideos • Porque en un paciente que utiliza corticosteroides de manera crónica se adelgaza y se ve fina la piel? • R = Debido al CATABOLISMO PROTEICO DE LAS PROTEÍNAS consistentes en perdidas de colágeno y resorción ósea. • Que alimentos se consideran bociogenos? • R = Mandioca, col de brucelas, repollo y coliflor
  • 20. PATOLOGIA DE TIROIDES • Cual es la causa mas frecuente de bocio en el mundo? • R = Déficit de yodo • Que es el signo de PEMBERTON EN BOCIO RETROESTERNAL? • R = Desfallecimiento, CONGESTIÓN FACIAL, OBSTRUCCIÓN VENOSA YUGULAR AL ELEVAR LOS BRAZOS POR ENCIMA DE LA CABEZA. • Que ocasiona el fenómeno de Jod-Basedow? • R = En bocio multinodular, AL APLICAR SUSTANCIAS CON YODO, SE PUEDE PROVOCAR HIPERFUNCIÓN DE ALGÚN NÓDULO. • Cual es el tratamiento más eficaz para revertir el estado hipertiroideo de un bocio multinodular toxico? • R = YODO RADIACTIVO • Cuales son los signos y síntomas en general del carcinoma tiroideo? 1) NÓDULO PALPABLE, INDOLORO ASINTOMÁTICO. 2) Estirpe Medular: BOCHORNOS, DIARREA Y SX DE CUSHING por producción de ACTH O CRH. • Que factores predisponen a carcinoma de tiroides? • R = Proteína cinasa del protoncogen RET y ANTECEDENTES DE RADIACIÓN EN CABEZA Y CUELLO.
  • 21. PATOLOGIA DE TIROIDES • Cual es la estirpe histológica mas común del carcinoma de tiroides y que lo caracteriza? 1) Carcinoma PAPILAR en 70-90%, 2) Asociado al GEN RET 3) El antecedente de RADIACIÓN EN CUELLO en la infancia se relaciona 4) Manifestándose como NÓDULO ÚNICO, 5) Donde la citología muestra CÉLULAS NUCLEARES SOBREPUESTAS CON APARIENCIA DE VIDRIO ESMERILADO y surcos longitudinales. 6) Mejor pronostico, • Con que enfermedades se relaciona el carcinoma papilar de tiroides? 1) Síndrome de GARDNER (poliposis adenomatosa familiar de colon) 2) Enfermedad de COWDEN (bocio familiar y hamartromas cutáneos) 3) Síndrome de TURCOT (tumores cerebrales mas poliposis en intestino grueso) • A que tipo de cáncer tiroideo son mas propensos los pacientes con enfermedad de Cowden? 1) FOLICULAR, AD, 2) Lo causa perdida de gen supresor de tumores. 3) Se caracteriza comúnmente por macrocefalia, HAMARTROMAS MÚLTIPLES, cáncer mamario o PÓLIPOS INTESTINALES.
  • 22. PATOLOGIA DE TIROIDES • Que distingue al carcinoma FOLICULAR tiroideo del papilar? 1) Que ESTA ENCAPSULADO, 2) INVADE VASOS SANGUÍNEOS. 3) Su VARIEDAD extensamente INVASIVA son las CÉLULAS DE HURTLE 4) Tiene MAL PRONÓSTICO. 5) Al extirparlo se hace SEGUMIENTO CON TIROGLOBULINA para descartar metástasis • Que origen tiene el CARCINOMA MEDULAR DE TIROIDES y que sustancia produce? 1) Se origina de las CÉLULAS C PARAFOLICULARES, 2) Secretor de CALCITONINA. 3) Secretor de ACTH 4) Asociado a mutación en oncogen RET, debido a eso SE DEBEN ESTUDIAR TAMBIÉN A FAMILIARES • Cual es el carcinoma de tiroides con peor pronostico? 1) ANAPLASICO con mortalidad del 95% en los primeros 6 meses. 2) Crecimiento rápido 3) Se han descrito supresión del gen supresor de tumores p53
  • 23. PATOLOGIA TIROIDES • Cual es el tratamiento de elección en carcinoma tiroideo? 1) Extirpación quirúrgica 2) Tratamiento medico con la INHIBICIÓN DE LA TSH DANDO LEVOTIROXINA A DOSIS ALTAS 3) Yodo radiactivo indicado en pacientes POSTOPERADOS DE CA TIROIDEO 4) Radioterapia externa: Solo en pacientes con EXTIRPACIÓN QUIRÚRGICA INCOMPLETA 5) Vigilancia clínica y por us
  • 24.
  • 25.
  • 26. SX DE CUSHING • Cual es la principal causa del Sx Cushing y su cuadro clínico? 1) Exógena por esteroides y 2da causa es por micro adenoma hipofisario 2) OBESIDAD CENTRAL 3) HAS por aumento del ANGIOTENSINOGENO HEPATICO 4) Intolerancia a la glucosa por incremento de la GLUCONEOGENESIS inducida por los glucocorticoides 5) FACIES DE LUNA LLENA 6) Trastornos menstruales 7) HIRSUTISMO • Como diagnosticas el sx de Cushing? 1) Inicialmente se debe demostrar HIPERCOLESTEROLISMO 2) Determinación de CORTISOL LIBRE URINARIO con sensibilidad de 90% 3) SUPRESIÓN CON DOSIS BAJAS DE DEXAMETASONA (prueba de NUGENT) 4) Cuando un paciente tiene un cortisol urinario y serico a las 8:00 am > 1.8 después de 1 mg/dl de dexametasona a las 23 hrs de la noche previa, el paciente tiene síndrome de Cushing.
  • 27. SX DE CUSHING • En cuanto al origen del síndrome de Cushing cuales son los estudios a realizar? 1) Se determina la hormona ADRENOCORTICOTROPICA plasmática, LOS VALORES POR DEBAJO DEL LIMITE DE DETECCIÓN CATALOGAN AL SÍNDROME COMO DE ORIGEN SUPRARRENAL. 2) La RM es la técnica de imagen de elección en el síndrome de Cushing de ORIGEN HIPOFISARIO. 3) La TAC es de elección para visualizar las GLÁNDULAS SUPRARRENALES. • En que consiste la PRUEBA DE NUGENT en el diagnostico de sx de Cushing? • R = SUPRESIÓN DE ACTH con administrar 1mg de dexametasona a las 23 Hrs y con nueva medición a las 8 AM. • En que consiste el tratamiento medico del sx de Cushing? • R = KETOCONAZOL, pues INHIBE LA SÍNTESIS DE CORTISOL pero se emplea por periodos cortos pues es hepatotoxico. • En que consiste el tratamiento quirúrgico del sx de Cushing? 1) CIRUGÍA HIPOFISARIA 2) RADIOTERAPIA HIPOFISARIA que ha mostrado ser efectiva en los casos que no se pueda realizar la CIRUGÍA TRANSESFENOIDAL
  • 28. ENFERMEDAD DE ADDISON • Cual es la principal causa de la enfermedad de Addison/insuficiencia suprarrenal? 1) 90% adrenalitis AUTOINMUNE, 2) En países subdesarrollados TB. • Cual es el cuadro clínico y DHE de la enfermedad de Addison? 1) HIPERPIGMENTACION de la piel (debido a que la deficiencia de cortisol, provoca un amento compensatorio de la PROPIOMELANOCORTINA que es precursora de la ACTH y de la hormona estimulante de melanocitos) 2) Vomito 3) Fatiga 4) Hipoglucemia 5) HIPOTENSIÓN 6) HIPONATREMIA 7) HIPERKALEMIA. • Cual es el desequilibrio hidroelectrolitico principal en la enfermedad de Addison? • R = ACIDOSIS METABOLICA con HIPERCALCEMIA.
  • 29. ENFERMEDAD DE ADDISON • Cuales son las manifestaciones clínicas de una CRISIS ADISONIANA o suprarrenal? 1) FIEBRE, 2) Deshidratación, 3) DOLOR ABDOMINAL 4) HIPOTENSIÓN O CHOQUE HIPOVOLEMICO, 5) Urea elevada 6) HIPONATREMIA, 7) HIPERKALEMIA, 8) HIPERCALCEMIA y 9) ACIDOSIS METABÓLICA. • Cual es el tratamiento de la crisis Adisoniana? • R = Administración inmediata de HIDROCORTISONA IV C-8 • Como realizas el diagnostico de enfermedad de Addison? 1) ESTANDAR DE ORO es la prueba de estrés con INSULINA IV la cual hace el diagnostico de adrenalitis secundaria, DONDE EL CORTISOL DEBE AUMENTAR, SINO ENTONCES SE HACE EL DIAGNOSTICO. 2) Medición de CORTISOL DURANTE LA MAÑANA y ADRENOCORTICOTROPINA con NIVELES BAJOS. 3) ACTH IM y MEDIR CORTISOL A LOS 60 MIN, EL CUAL DEBE AUMENTAR. 4) En los pacientes con adrenalitis autoinmune se encuentran AUTOANTICUERPOS ANTI-21HIDROXILASA. • Cual es el manejo de la enfermedad de Addison? • R = Hidrocortisona o PREDNISONA.
  • 30. SINDROME DE WATERHOUSE- FRIEDERICHSEN • Como se le llama al síndrome que sucede cuando una HEMORRAGIA SUPRARRENAL MASIVA complica una infección bacteriana y cual es el principal agente etiológico? • R = Se le conoce como SINDROME DE WATERHOUSE- FRIEDERICHSEN, frecuente en niños y su principal agente etiológico es la SEPTICEMIA OCASIONADA POR NEISERIA MENINGITIDIS • Que consecuencias clínicas tiene el síndrome de watherhouse-Friderichsen? 1) HIPOTENSION rápidamente progresiva que acaba en shock 2) CID con purpura generalizada 3) INSUFICIENCIA CORTICOSUPRARENAL
  • 31. HIPERALDOSTERONISMO • Cual es la etiología del hiperaldosteronismo? 1) Primario: Hipersecreción de aldosterona la cual NO SE SUPRIME EN RESPUESTA A LA EXPANSIÓN DE VOLUMEN. 2) Secundario: Altos niveles de renina la cual se reprime en respuesta a la expansión de volumen. • Cuales son los datos clínicos del hiperaldosteronismo? 1) Puede cursar asintomático o 2) Cefalea 3) HAS 4) DEBILIDAD MUSCULAR 5) PARESTESIAS 6) Tetania 7) Parálisis • Cuales son los datos de laboratorio encontrados en Hiperaldosteronismo primario? • R = HIPERNATREMIA, HIPOKALEMIA y ALCALOSIS METABÓLICA • -A 30-year-old man is having symtoms of muscle weakness, fatigue, and headaches. On examination, his blood pressure is 180/100 mm Hg, pulse 80/min, JVP 4 cm, heart sounds normal, and lungs clear. His serum potassium level is 2.5 mEq/L and bicarbonate 30 mEq/L. The urine potassium is 40 mEq/L. Plasma renin is low and aldosterone is high. • -Primary hyperaldosteronism is characterized by hypertension with high plasma aldosterone and low plasma rennin
  • 32. HIPERALDOSTERONISMO • Como diagnosticas hiperaldosteronismo? • R = ELEVACIÓN DE ALDOSTERONA PLASMÁTICA POSTERIOR A LA INGESTIÓN DE CLORURO DE SODIO por cada comida durante dos a tres días o un NIVEL DE ALDOSTERONA EN PLASMA ALTO DESPUÉS DE LA INFUSIÓN INTRAVENOSA DE SOLUCIÓN SALINA (1 lt x hr durante 4 hrs). • Cual es el manejo del hiperaldosteronismo? 1) CIRUGÍA A PACIENTES CON ALDOSTERONOMA con adrenalectomía laparoscópica 2) En el caso de HIPERPLASIA SUPRARRENAL BILATERAL IDIOPÁTICA se debe recibir TRATAMIENTO MEDICO CON ESPIRONOLACTONA 3) En el caso de hiperaldosteronismo tratable con esteroides se utiliza la DEXAMETASONA para mantener una supresión del eje hipotálamo hipófisis. • Cuales son los datos que caracterizan al síndrome de Conn en hiperaldosteronismo? • R = Hipotensión, hipokalemia, aumento de aldosterona y supresión secundaria de renina.
  • 33. HIPOALDOSTERONISMO • Cual es la etiología del hipoaldosteronismo? 1) Puede producirse por un estimulo insuficiente por parte de la renina (HIPORRENINEMICO) “NEFROPATÍA DIABÉTICA” 2) Por un fallo primario en la secreción de aldosterona (HIPERRENINEMICO) “ENFERMEDAD DE ADDISON” 3) Por resistencia a la acción de la aldosterona (SEUDOHIPOALDOSTERONISMO). 4) Algunos medicamentos pueden ocasionarlo como los AINES, heparina (suprime la secreción de renina), IECAS. • Cual es el cuadro clínico del hipoaldosteronismo? 1) CALAMBRES 2) Debilidad muscular • Cuales son los datos de laboratorio del hipoaldosteronismo? 1) HIPERKALEMIA 2) HIPONATREMIA 3) ACIDOSIS METABOLICA HIPERCLOREMICA 4) Deshidratación 5) Se debe hacer diagnostico diferencial de enfermedad de Addison
  • 34. HIPOALDOSTERONISMO • Como se diagnostica el hipoaldosteronismo? 1) Analíticamente se deben obtener los niveles plasmáticos de aldosterona, cortisol y actividad de renina 2) Si se sospecha de enfermedad de ADDISON SE DEBE REALIZAR LA PRUEBA DE ESTIMULACIÓN CON HORMONA ACTH. 3) Si existe aldosterona baja con actividad de renina (HIPORRENINEMICO), debemos pensar en NEFROPATIA DIABETICA, 4) Si obtenemos aldosterona baja y renina alta (HIPERRENINEMICO), debemos sospechar ADISON o una forma grave de hiperplasia suprarrenal congénita. 5) Si lo que nos reportan es ALDOSTERONA ELEVADA Y ACTIVIDAD DE RENINA ALTA, debemos pensar en SEUDOHIPOALDOSTERONISMO. • Cual es el manejo del hipoaldosteronismo 1) INGESTIÓN LIBRE DE SAL como TRATAMIENTO MEDICO no farmacológico 2) El tratamiento medico CON FÁRMACOS es con FLUDROCORTISONA.
  • 35. ALTERACIONES POR DHE, Na y K • ENFERMEDAD DE ADDISON: • Acidosis Metabolica con hipercalcemia • DHE: Hiponatremia – Hiperkalemia • Cual es el manejo de la enfermedad de Addison? • R = Hidrocortisona o PREDNISONA. • HIPOALDOSTERONISMO: • Acidosis Metabolica Hipercloremica • DHE: Hiponatremia – Hiperkalemia • Cual es el manejo del hipoaldosteronismo 1) INGESTIÓN LIBRE DE SAL como TRATAMIENTO MEDICO no farmacológico 2) El tratamiento medico CON FÁRMACOS es con FLUDROCORTISONA. • HIPERALDOSTERONISMO: • Alcalosis Metabolica • DHE: Hipernatremia - Hipokalemia • Cual es el manejo del hiperaldosteronismo? 1) CIRUGÍA A PACIENTES CON ALDOSTERONOMA con adrenalectomía laparoscópica 2) En el caso de HIPERPLASIA SUPRARRENAL BILATERAL IDIOPÁTICA se debe recibir TRATAMIENTO MEDICO CON ESPIRONOLACTONA 3) En el caso de hiperaldosteronismo tratable con esteroides se utiliza la DEXAMETASONA para mantener una supresión del eje hipotálamo hipófisis.
  • 36. HIPERCALCEMIA • Cuales son los eventos metabólicos implicados en la regulación del calcio? 1) En presencia de hipercalcemia se estimula la secreción de CALCITONINA, la cual INHIBE LA OSTEOLISIS y ESTIMULA LA CALCIURIA. 2) Cuando la hipercalcemia permanece, la CALCITONINA INHIBE LA REABSORCIÓN OSTEOCLASTICA, LA SECRECIÓN DE PARATHORMONA Y LA SÍNTESIS DE VITAMINA D. • Cual es la etiología de la hipercalcemia? 1) HIPERPARATIROIDISMO PRIMARIO: Puede deberse a ADENOMA ÚNICO o HIPERTROFIA GLANDULAR. Se caracteriza por ELEVACIÓN DE LA PTH, HIPERCALCEMIA E HIPOFOSFATEMIA. 2) Hiperparatiroidismo secundario: Puede deberse a IR, sx de mal absorción y raquitismo. 3) El adenoma y las neoplasias malignas causan 90% de las hipercalcemias. 4) Es frecuente en el CÁNCER DE MAMA. • Cual es el cuadro clínico de la hipercalcemia? 1) Ca entre 10.5-12 g/dL es asintomático 2) Niveles mayores forman TRIADA CLÁSICA DE ESTREÑIMIENTO, NAUSEA Y VOMITO. 3) Fatiga, letargo, cefalea, sed, nausea, vomito, NEFROLITIASIS, 4) ACORTAMIENTO DEL INTERVALO QT EN EL EKG. • Cual es el manejo de la hipercalcemia? 1) HIDRATACION 2) FUROSEMIDE es eficaz para evitar la reabsorción tubular de calcio • Cual es el fármaco que se indica en hipercalcemia crónica? • R = TIAZIDAS pues previenen cálculos renales.
  • 37. HIPOCALCEMIA • Cual es la etiología de la hipocalcemia? 1) Ca sérico menor 8.5. Previamente se debe confirmar que la albumina se encuentre en niveles séricos normales, ya que el descenso de 1 g/dL de albumina se acompaña de un descenso de 0.8 mg/dL de calcio. 2) La causa mas común de HIPOPARATIROIDISMO ES LA CIRUGÍA TIROIDEA Y LA SEGUNDA ES LA RADIACIÓN. 3) La hipomagnesemia < 1 mg/dL puede producir hipocalcemia 4) La hipermagnesmia reduce la secreción de PTH. 5) La hiperfosfatemia causa hipercalcemia por formación de complejos con el calcio. 6) En pacientes transfundidos se produce hipocalcemia ya que el citrato del paquete globular actúa como paquete globular. 7) EL SINDROME DE DIGEORGE se caracteriza por AUSENCIA DE GLÁNDULAS PARATORIDEAS, con DÉFICIT DE PTH, ASOCIADO A DISPLASIA TIMICA, y malformaciones cardiacas por alteración en el desarrollo de LAS TERCERAS Y CUARTAS BOLSAS FARÍNGEAS. 8) Las formas adultas de hipoparatiroidismo se asocian a enfermedades autoinmunes como insuficiencia suprarrenal, anemia perniciosa e hipotiroidismo. 9) En la PANCREATITIS, el calcio se deposita en áreas con necrosis grasa. 10) EL SINDROME DE HUESO HAMBRIENTO se presenta en paratiroidectomia. 11) En enfermedades del APARATO DIGESTIVO se favorece el déficit de vitamina D por disminución en la absorción. 12) La DIFENILHIDANTOINA inhibe la absorción de calcio. • Cual es el cuadro clínico de la hipocalcemia? 1) HIPEREXITABILIDAD neuromuscular. 2) Parestesias, HIPERREFLEXIA, ESPASMO CARPOPEDAL, irritavilidad, SIGNO DE CHEVOSTEK Y TROSSEAU. 3) El EKG MUESTRA PROLONGACIÓN DEL ST.
  • 38. OSTEOPOROSIS • Cual es el manejo de la osteoporosis? 1) ALENDRONATO, RISENDRONATO. 2) PTH a dosis bajas, al igual que los bifosfonatos han demostrado aumento en la masa ósea, 3) Terapia de reemplazo hormonal con estrógenos previene fracturas vertebrales. 4) RALOXIFENO Y TAMOXIFENO ya que AUMENTAN LA DENSIDAD ÓSEA, DISMINUYEN RIESGO DE FRACTURAS VERTEBRALES Y EL CÁNCER DE MAMA 5) La TIBOLONA es una sustancia sintética con acción tejido específica que tiene efecto ESTROGÉNICO, PROGESTACIONAL Y ANDROGÉNICO según el lugar de acción. Esta molécula mejora los síntomas vasomotores, el estado anímico, la libido, la atrofia urogenital y previene la osteoporosis postmenopáusica; no produce mastalgia ni modifica la densidad mamográfica. Se observan además efectos beneficiosos sobre algunos parámetros cardiovasculares y hemostáticos. EN EL ENDOMETRIO, LA TIBOLONA SE CONVIERTE EN SU METABOLITO PROGESTAGÉNICO/ANDROGÉNICO Y EN CONSECUENCIA NO PRODUCE ESTIMULACIÓN ENDOMETRIAL NI SE NECESITA AGREGAR UN PROGESTÁGENO. Esta ventaja mejora el cumplimiento de la terapia hormonal de reemplazo (THR) debido que no provoca sangrados por disrupción
  • 40. OSTEOMALACIA • Cual es la etiología de la osteomalacia? • R = Déficit de VITAMINA D EN ADULTOS • Cual es el cuadro clínico de la osteomalacia? 1. DEBILIDAD MUSCULAR PROXIMAL, asociada con desgaste muscular, hipotonía y dificultad para la movilización. 2. EL OSTEOCOPO (dolor óseo), es mas notorio en LA REGION LUMBAR, PELVIS Y EXTREMIDADES PELVICAS DONDE SE PRESENTAN FRACTURAS. 3. Las fracturas suceden con un mínimo de traumatismo. 4. Los síntomas deben incluir: DOLOR OSEO, DEFORMIDADES DEL ESQUELETO Y/O CRANEO, piernas arqueadas o rodillas malformadas, PECHO EN PALOMA, Deformaciones dentales. • Como se diagnostica la osteomalacia? 1. MARCLAJE CON TETRACICLINAS. Este medicamento se deposita en forma de bandas en el sitio de mineralización y al ser fluorescentes pueden ser visualizados al microscopio. La tasa de crecimiento del esqueleto puede estimarse en las biopsias de las crestas iliacas al medir la distancia entre las bandas de tetraciclina. Si disminuye la distancia entre las bandas se hace el diagnostico. 2. EL HALLAZGO RADIOGRÁFICO CARACTERÍSTICO ES LA REDUCCIÓN EN LA DENSIDAD ÓSEA CON ADELGAZAMIENTO DE LA CORTEZA. 3. Laboratorio: Los pacientes con déficit de vitamina D se presentan con HIPOFOSFATEMIA, CALCIO SÉRICO BAJO E HIPOPARATIROIDISMO SECUNDARIO. • Cual es el manejo de la osteomalacia? • R = Calcitriol o CALCIO + VITAMINA D en combinación.
  • 41. DEFICIT DE ADH • Que causa la deficiencia de ADH? • R = Diabetes insípida central con POLIURIA Y POLIDIPSIA, causando HIPONATREMIA con la ingesta inadecuada de líquidos • -A 27-year-old woman presents because she is feeling unwell. Her examination is normal, but her serum sodium is 115 mEq/L. Her urine lytes: sodium 55 mEq/L, osmolality 100 mOsm/L. This condition causes the greatest amount of medullary washout: • -Primary áternn can cause greatermedullary washout át either nephrogenic or central DI because primary áternn tends to cause áternn of the ECF átern. This tends to increase total delivery of sodium chloride and áter to the inner medulla. It also increases renal blood flow, and increased flow through the vasa recta reduces ability to trap solutes in the medulla.
  • 42. DIABETES INSIPIDA CENTRAL • A que se debe la diabetes insípida central primaria y secundaria? 1) Primaria : Ac vs ARGININA-VASOPRESINA. 2) LESIÓN DEL HIPOTÁLAMO, del tallo hipofisario o por cualquier CAUSA INFECCIOSA, TRAUMÁTICA etc.. • Cuales son las causas de DI central y cuales son los hallazgos en la TAC? • R = DI IS USUALLY CAUSED BY DESTRUCTION, OR AGENESIS, OF THE POSTERIOR PITUITARY, its normal signaling is lost. PITUITARY DI CAN ALSO RESULT FROM TRAUMA, TUMORS (BOTH PRIMARY AND SECONDARY), granulomas, infections, inflammatory diseases, chemical toxins, congenital malformations, and genetic disorders. Depending on the cause, the MRI may demonstrate other associated findings. • Que causa diabetes insípida en el ultimo trimestre de embarazo-puerperio? • R = Es inducido por VASOPRESINA en el ultimo trimestre de embarazo, relacionada frecuentemente con oligohidramnios, preeclampsia o disfunción hepática. RESPONDE A DESMOPRESINA
  • 43. DIABETES INSIPIDA NEFROGENICA • Que causa y cual es el cc de la diabetes insípida nefrogenica? 1) DEFECTO TUBULORENAL DE LOS RECEPTORES DE LA ADH que interfiere con la reabsorción de agua. 2) POLIURIA QUE NO RESPONDE A VASOPRESINA, con secreción normal vasopresina. 3) Congénita: Desde el nacimiento y se debe a expresión defectuosa de los RECEPTORES RENALES DE VASOPRESINA V2. • -A 38-year-old man is admitted to hospital for investigation of polyuria and a low serum sodium concentration. After a fluid deprivation test, there was no change in his urine osmolality. He was then given vasopressin (DDAVP) with no change in urine volume or urine osmolality after 2 hours. • -There is little or no response to vasopressin after fluid deprivation in complete nephrogenic DI. Incomplete nephrogenic DI will show some response. • -A19-year-old man and one of his two brothers has had polyuria and polydipsia since birth. Neither his sisters nor his parents are affected. • -Nephrogenic DI can be inherited on the X chromosome. Its X-linked recessive nature means that males are predominantly affected. Only women who are homozygous are affected.
  • 44. DIABETES INSIPIDA NEFROGENICA • Como diagnosticas DM insípida? 1) JUICIO CLÍNICO, no hay prueba de laboratorio contundente. 2) RECOLECCIÓN DE ORINA DE 24 HRS MIDIENDO VOLUMEN <2LTS/24 HRS SIN HIPERNATREMIA. 3) Desmopresina: MIDIENDO VOLUMEN URINARIO 12 HRS ANTES Y 12 HRS DESPUÉS, EN EL CUAL LOS PACIENTES CON DIABETES INSÍPIDA CENTRAL HABRÁ DISMINUCIÓN DE SED Y POLIURIA, 4) Realizándose IRM analizando hipófisis e hipotálamo, se observa saco hipofisario engrosado: Células de langerhans, sarcoidosis, hipofisitis linfocitica, Histiocitosis (proliferación de macrófagos en diferentes aparatos y sistemas). 5) Desmopresina en DM nefrogenica valorando su concentración con restricción de líquidos aumentando sus valores. • Cual es el manejo de la DM insípida nefrogenica o central? 1) CENTRAL: DESMOPRESINA tmb se utiliza en DM INSÍPIDA RELACIONADA CON EMBARAZO o puerperio, evitar factores de agravamiento como administración de glucocorticoides. 2) NEFROGENICA: HIDROCLOROTIAZIDA con suplemento de potasio, INDOMETACINA en fase aguda.
  • 45. ACROMEGALIA• Cual es la causa principal de acromegalia? • R = ADENOMA HIPOFISIARIO, también por secreción ectópica de GnRH o GH por tumores carcinoides hipotalámicos, bronquiales o pancreáticos. • Que características forman parte del SÍNDROME DE CARNEY? • R = MIXOMA AURICULAR, NEUROMA DEL ACÚSTICO Y PIGMENTACIÓN PUNTEADA DE LA PIEL • Cuales son los signos y síntomas de la acromegalia o gigantismo? 1) ESTATURA ELEVADA, gigantismo antes del cierre hiposifiario. 2) Acromegalia significa crecimiento de extremidades. 3) SÍNDROME DEL TÚNEL CARPIANO. 4) RASGOS FACIALES TOSCOS, PROGNATISMO CON MALA OCLUSIÓN. 5) HAS 50%. DM 30% • Cuales son los lab para dx acromegalia o gigantismo? 1) AYUNO. > IGF-1 5 VECES DE LO NORMAL. 2) Prolactina aumentada algunas veces. 3) Glucosa: la DM ES COMÚN. 4) Calcio: Para descartar hiperparatiroidismo. 5) Fosforo: Se encuentra aumentada. 6) T4 y TSH: HIPOTIROIDISMO COMÚN. 7) SE RECOMIENDA ADMINISTRAR UNA SOLUCIÓN DE DEXTROSA MIDIENDO LA GH 1 HR DESPUÉS, SI LOS VALORES ESTÁN < O NORMALES AL IGUAL QUE LA IGF-1, YA QUE ESTA HORMONA RESPONDE CON DISMINUCIÓN A LA INGESTA DE GLUCOSA. • Que medicamentos se utilizan en caso de no remitir > GH o IGF-1 con adenectomía transesfenoidal hipofisiaria? 1) ANÁLOGOS DE LA SOMATOSTATINA COMO OCTREOTIDE para conservar disminución del tamaño del adenoma. GH sérica 1 mg y 2.5 mg, pero no responde bien si previos al tratamiento la GH > 20 ng/ml. 2) PEGVISOMANT es un antagonista del receptor de la GH bloqueando sus efectos, vía SC, pero no reduce el tamaño de los adenomas. 3) RADIOCIRUGÍA: en los pacientes no responden al tratamiento quirúrgico o medico, no se realiza si hay extensión suprecelar por que puede dañar el quiasma óptico.
  • 46. NEM I • Cuales son las características de la NEM I O SÍNDROME DE WERMER? 1) Se caracteriza por neoplasias en PARATIROIDES, PÁNCREAS y ADENOHIPOFISIS. 2) Se hereda de forma AD en el CROMOSOMA 11 • Cual es la clínica del NEM I o Sindrome de Wermer? 1) HIPERPARATIROIDISMO PRIMARIO: Que puede manifestarse con MIALGIAS, DEBILIDAD, HIPERCALCEMIA, ALTERACIONES MENTALES, NEFROLITIASIS Y ANORMALIDADES ÓSEAS. La hipercalcemia puede provocar aumento de la secreción de gastrina y un Sx Zollinger-Ellison secundario. EL TRATAMIENTO CONSISTE EN LA EXTIRPACIÓN QUIRÚRGICA DE LAS PARATIROIDES. 2) TUMORES DE LAS CÉLULAS DE LOS ISLOTES PANCREÁTICOS: Los GASTRINOMAS SON LA CAUSA MAS IMPORTANTE de morbimortalidad y la mayoría son multiples. Los insulinomas son generalmente benignos y únicos. 3) NEOPLASIA DE LA ADENOHIPOFISIS: La neoplasia MAS FRECUENTE ES EL PROLACTINOMA seguida del tumor secretor de hormona del crecimiento que causa acromegalia, pueden presentarse como enfermedad de Cushing secundaria a adenoma hipofisario. • Como realizas el diagnostico de NEM I? 1) Con 2 de las 3 neoplasias características del síndrome 2) El MEN I familiar se define como la presencia de un paciente afectado con esta enfermedad y un familiar de primer grado que presente al menos una de las tres neoplasias características • Cual es el manejo del NEM I? 1) CIRUGÍA 2) PARA CADA MANIFESTACIÓN de sobre producción hormonal se da el medicamento individual
  • 47. MEN II • Cual es la etiología cromosómica del MEN II? • R = Se hereda de forma AD, en el CROMOSOMA 10. • Cual es la clasificación del MEN II?  MEN II A o Síndrome de Sipple: Con la presencia de cáncer medular de tiroides, FEOCROMOCITOMA e hiperparatiroidismo primario.  MEN II B: Con la presencia de cáncer medular de tiroides, feocromocitoma, NEUROMAS MUCOSOS, ganglioneuromatosis intestinal y HABITO MARFANOIDE. • Como diagnosticas el MEN II? 1) 2 a 3 tumores de los antes mencionados 2) La mutación del protoncogen RET esta indicada en todos los familiares de primer grado. 3) EL CARCINOMA MEDULAR DE TIROIDES SUELE SER LA PRINCIPAL CAUSA DE MUERTE.
  • 48.
  • 49. GASTRINOMA • Cuales son los tumores de las células de los islotes pancreáticos? • R = Gastrinoma, insulinoma, glucagonoma, vipoma y somatostinoma. GIGVS • Cual es el cuadro clínico del gastrinoma? 1) Enfermedad ulcerosa con dolor abdominal 2) Diarrea 3) ERGE. 4) Se debe sospechar en ULCERAS PÉPTICAS MÚLTIPLES. • Como diagnosticas el gastrinoma? 1) Los niveles séricos de GASTRINA EN AYUNO MAYORES DE 1000 Y PH < 2.5 EN LA SECRECIÓN GÁSTRICA HACEN EL DIAGNOSTICO DE GASTRINOMA. 2) Se debe SUSPENDER LA ADMINISTRACIÓN DE ANTAGONISTAS DE H2 UNA SEMANA ANTES Y DE OMEPRAZOL. 3) En paciente con niveles séricos menores a 1000 y PH < 2.5 se debe pensar en H. Pilory 4) La aclorhidria por gastritis atrófica o anemia perniciosa es una de las causas mas frecuentes de hipergastrinemia. • Cual es el manejo del gastrinoma? 1) Para la hipersecreción acida OMEPRAZOL 2) Gastrinoma RESECCIÓN QUIRÚRGICA PARA EVITAR METÁSTASIS
  • 50. INSULINOMA• Cual es el cuadro clínico del insulinoma? 1) SECUNDARIOS AL EFECTO HIPOGLUCEMICO como cefalea, mareo, debilidad, confusión, desorientación y coma. 2) Síntomas secundarios al aumento de catecolaminas circulantes provocado por la hipoglucemia como sudación, temblor y palpitación. 3) La mayoría de las crisis se relaciona con el ayuno. • Como realizas el diagnostico de insulinoma? 1) Se basa en la TRIADA DE WIPPLE la que consiste en síntomas de neuroglucopenia, niveles de glucosa en sangre por debajo de 45 y mejoría inmediata de los síntomas después de la administración de glucosa 2) Si se sospecha de insulinoma se debe realizar una PRUEBA DE AYUNO DURANTE 72 HRS, considerado como el método diagnostico de elección, donde se miden los niveles de glucosa, péptido C e insulina cada 6 hrs y siempre que exista clínica de hipoglucemia la prueba se suspende si aparece una glucemia inferior a 45 o si se presentan síntomas neuroglucopenicos. En sujetos normales la insulina debe disminuir en presencia de hipoglucemia. Los criterios diagnósticos de insulinoma incluyen insulina plasmática mayor de 6 cuando la glucemia es menor de 40, proinsulina > .2, péptido C > 200 y cociente de insulina/glucosa > .3. 3) Existen AINTICUERPOS ANTIINSULINA Y NIVELES ELEVEADOS DE PROINSULINA. • Cual es el manejo del insulinoma? 1) Para las crisis de hipoglucemia se realizan comidas frecuentes y de escasa cantidad 2) En el caso de hipoglucemia se administra 50 ml de dextrosa al 50%, si no hay respuesta dentro de los primeros 15 minutos, la dosis puede repetirse, en caso de que no se pueda canalizar al paciente se administra 1 mg de guagón por vía IV, IM o subcutánea. 3) El DIAZOXIDO inhibe la secreción de insulina, pero provoca retención de líquidos e hirsutismo con frecuencia. 4) La resección completa del tumor cura a la mayoría de los pacientes. Cuando no se logra localizar el tumor se realiza una pancreatectomia distal gradual hasta localizarlo dejando por lo menos 20-30% del páncreas hasta localizarlo. 5) EN EL CASO DE TUMOR MALIGNO CON METÁSTASIS O TUMOR IRRESECABLE SE RECOMIENDA EL TRATAMIENTO A LARGO PLAZO CON DIAZOXIDO, SI NO RESPONDE SE INICIA QUIMIOEMBOLIZACION.
  • 51. VIPOMAS• Que son los vipomas? • R = Son tumores derivados de las CÉLULAS DE LOS ISLOTES DEL PÁNCREAS que producen grandes cantidades de PEPTIDO INTESTINAL VASOACTIVO, el cual AUMENTA LA CONTRACCIÓN DEL MUSCULO LISO, INHIBE LA SECRESION GÁSTRICA Y TIENE EFECTOS VASODILATADORES. • Cual es el cuadro clínico asociado a vipomas? 1) El síntoma predominante es una DIARREA ACUOSA, que produce deshidratación, hipokalemia y debilidad. 2) La DIARREA ES SECRETORA, persiste durante el ayuno y NO RESPONDE A ANTIDIARREICOS. 3) Acidosis metabólica • Como realizas el diagnostico del vipoma? 1) Presencia de DIARREA SECRETORA 2) Niveles elevado de PÉPTIDO INTESTINAL VASOACTIVO 3) TUMORACION pancreática • Como se maneja el vipoma? 1) Corregir la deshidratación y desequilibrio hidroelectrolitico, en especial de la HIPOKALEMIA. 2) Resección del tumor y en presencia de metástasis se reduce primero el tumor. 3) Si no se encuentra el tumor se busca en las glándulas suprarrenales y las cadenas simpáticas, en caso de no identificarse en ninguno de estos sitios se realiza resección del 80% del tumor. 4) En el caso de tumor maligno inoperable o presencia de metástasis se recomienda el uso del análogo de la somatostatina de acción prolongada, también puede utilizarse la embolizacion de la arteria hepática.
  • 52. GLUCAGONOMAS • Que son los glucagonomas? 1) Son tumores derivados de las CELULAS ALFA DE LOS ISLOTES DEL PANCREAS los cuales producen una gran cantidad de la HORMONA GLUCAGON y puede secretar otros péptidos (polipeptido pancreático, gastrina y somatostatina). 2) Es mas frecuente en mujeres PERI O POSTMENOPAUSICAS • Cual es el cuadro clínico de los glucagonomas? 1) DERMATITIS NECROLITICA MIGRATORIA es característica de los glucagonomas. Inicia como una zona eritematosa y escamosa en la ingle y glúteo, se extiende al perineo y extremidades inferiores, en ocasiones hay lesiones peri orales, posteriormente se hace ampolloso y deja erosiones cuando las ampollas se rompen. 2) Intolerancia a la glucosa o DIABETES 3) Anemia 4) QUEILITIS Y ESTOMATITIS 5) Diarrea 6) Hipercolesterolemia • Como diagnosticas los glucagonomas? 1) Con el cuadro clínico y se confirma al encontrar niveles plasmáticos de GLUCAGON EN AYUNO > 1000 (NORMAL 150) Y UNA TUMORACIÓN PANCREÁTICA. 2) NO SE SUPRIME LA LIBERACIÓN DEL GLUCAGON CON LA ADMINISTRACIÓN DE GLUCOSA Y TAMPOCO SE ESTIMULA CON LA ADMINISTRACIÓN DE ARGININA. • Cual es el manejo del glucagonoma? 1) Debido a la presencia de METASTASIS EN LA MAYORIA DE LOS PACIENTES, no es posible una resección quirúrgica curativa, pero en estos casos se extirpa el tumor primario y se reduce el volumen de las metástasis. 2) Los SUPLEMENTOS DE ZINC mejoran la dermatitis 3) Los ANALOGOS DE LA SOMATOSTATINA normalizan los valores de glucagon y aminoácidos séricos, mejoran la dermatitis y la diarrea, promueven el aumento de peso, pero NO corrigen la intolerancia a la glucosa. 4) La quimioterapia NO es útil pero cuando se decide utilizarla, la combinación que ha demostrado mas eficacia es la de ESTREPTOZOCINA +DACARBACINA. 5) EL 50% DE LAS MUERTES ES CAUSADO POR EVENTOS TROMBOEMBOLICOS.
  • 53. SOMATOSTINOMAS • Que son los somatostatinomas? 1) Son tumores derivados de las CELULAS DELTA DEL PANCREAS productores de somatostatina, al igual que los VIPomas y glucagonomas 2) 70% SON MALIGNOS 3) La somatostatina es un inhibidor potente de la secreción gástrica y pancreática • Cual es el cuadro clínico de los somatostatinomas? 1) Es causado por la inhibición de la secreción tanto exocrina como endocrina del páncreas 2) LA TRIADA CLASICA CONSISTE EN DIABETES, DIARREA-ESTEATORREA Y COLELITIASIS. • Como diagnosticas los somatostatinomas? • R = La mayoría se localiza incidentalmente al realizar cirugías abdominales o endoscopias. EL diagnostico se realiza al demostrar la somatostatina elevada en plasma. • Cual es el manejo de los somatostatinamas? 1) ANÁLOGOS DE LA SOMATOSTATINA con el fin de inhibir la activación de los receptores por parte de la somatostatina secretada por el tumor. 2) EXTIRPACIÓN QUIRÚRGICA si se localiza el tumor 3) Para enfermedad metastasica se usa QUIMIOTERAPIA
  • 54. FEOCROMOCITOMA • Cual es la etiología del feocromocitoma? 1) Tumores de origen NEUROECTODERMICO desarrollado a partir de las CÉLULAS CROMAFINES DEL SNS (ganglios simpáticos y medula suprarrenal) productores de CATECOLAMINAS y la mayoría de ellos libera epinefrina y norepinefrina. 2) Los feocromocitomas ASOCIADOS A NEM SECRETAN SOLO EPINEFRINA. • Cual es la clasificación y localización mas común de los feocromocitomas? 1) Feocromocitomas si provienen de las células cromafines y paragangliomas si son de origen extra renal. 2) La localización mas frecuente es la adrenal 85%, siempre es unilateral, en caso de ser bilateral debe sospecharse de NEM • Cual es el cuadro clínico del feocromocitoma? 1) HAS 90% 2) Crisis adrenal caracterizada por triada clásica de: CEFALEA, PALPITACIONES Y SUDORACIÓN. • Cuales son los métodos diagnósticos de feocromocitoma? 1) Demostración de niveles elevados de catecolaminas o sus metabolitos en sangre o en orina, lo que establece el diagnostico. 2) Se determina el nivel de catecolamina libre o no metabolizadas (epinefrina y norepinefrina) o sus metabolitos ( metanefrina, normetanefrina y ACIDO VANILMANDELICO) en orina de 24 hrs. • Cuales son los métodos de imagen para dx de feocromocitoma? 1) TAC 2) Gamagrafia para localizar paragangliomas • Cual es el manejo del feocromocitoma? 1) Cirugía, pero antes de realizarla es necesario realizar un bloqueo alfadrenergico, generalmente se utiliza FENOXIBENZAMINA durante 10-14d previos a la cirugía. 2) Durante la anestesia o manipulación del tumor SUELE APARECER HIPERTENSIÓN Y ARRITMIAS, LA PRIMERA SE TRATA CON FENTOLAMINA Y LA SEGUNDA CON PROPANOLOL.
  • 55. TUMORES CARCINOIDES • Cual es la etiología de los tumores carcinoides? • R = Provienen de las CELULAS NEUROENDOCRINAS, en su citoplasma hay gránulos llenos de hormonas (serotonina, histamina, dopamina, prostaglandinas. • Desde su origen embriológico, donde se localizan los carcinoides del intestino anterior, medio y posterior? 1) Anterior: Pulmones, bronquios y estomago 2) Medio: Intestino delgado, apéndice o colon proximal. 3) Posterior: Colon distal y recto. • Cual es la localización más común del tumor carcinoide? • R = APÉNDICE, luego recto, íleon, pulmón, bronquios y estomago.
  • 56. TUMOR CARCINOIDE • Cual es el cuadro clínico producido por el tumor carcinoide? • R = Producen síntomas inespecíficos, vagos, relacionados con el órgano afectado tales como: 1) Carcinoide apendicular: El la LOCALIZACIÓN MAS FRECUENTE. Si el tumor mide mas de 2 cm se debe realizar una hemicolectomia derecha, si mide entre 1-2 cm e invade la serosa se debe realizar una hemicolectomia derecha y linfadenectomia regional. LA APENDICETOMÍA SE CONSIDERA CURATIVA CUANDO MIDEN MENOS DE 1 CM. 2) Carcinoide rectal: Estos secretan mas glucagon mas que serotonina. Puede manifestarse como SANGRADO RECTAL, DOLOR O CONSTIPACION. Los tumores mayores de > 2 cm se tratan con una resección anterior baja o una abdominoperineal. 3) Carcinoide del intestino delgado: La mayoría se encuentra el ILEON. Pueden provocar desde molestias abdominales leves hasta dolor abdominal intermitente causado por la obstrucción intestinal, la cual puede ser secundaria a la fibrosis que producen estos tumores o con menor frecuencia se manifiestan con diarrea, hemorragia por ulceración de la mucosa o intususcepción. SON LA CAUSA MAS FRECEUNTE DE SX CARCINOIDE. 4) Carcinoides pulmonares y bronquiales: Neumonías recurrentes, tos hemoptisis, dolor pleurítico. La resección en cuna o segmentaria se recomienda para los tumores localizados en caso de los carcinoides atípicos se utilizan resecciones mas amplias. 5) Carcinoides gástricos: Se originan de las células enterocromafines de la mucosa gástrica, se manifiestan como anemia o dolor abdominal. EL tipo 1 y mas frecuente en mujeres se asocia a hipergastrinemia y gastritis atrófica. 6) Carcinoides del colon: La mayoría en COLON DERECHO, siendo el CIEGO el sitio mas afectado. Pueden producir dolor, anorexia y perdida de peso. • Como diagnosticas los tumores carcinoides? 1) Cuadro clínico 2) Concentración de serotonina o de sus metabolitos en orina 3) Imagen 4) Se confirma por biopsia 5) Medición del ACIDO 5-HIDROXINDOLACETICO en una muestra de ORINA de 24 hrs, el cual si es mayor de 10 mg indica EXCESO DE SEROTONINA. 6) En SANGRE se miden los niveles de CROMOGRANINA A, glucoproteina producida por las células derivadas de la cresta neural y secretada en los tumores neuroendocrinos, relacionada directamente con el volumen tumoral
  • 57. TUMOR CARCINOIDE • Cual es el manejo del tumor carcinoide? 1) Depende de la localización del tumor y la presencia de metástasis 2) Sin metástasis la cirugía es curativa 3) Cuando hay metástasis por lo general es tratamiento paliativo 4) Quimioterapia a base de cisplatino y etoposido • Cual es el pronostico del tumor carcinoide? • R = Depende de la localización, tamaño y metástasis. El mejor pronóstico lo tienen los carcinoides bronquiales y apendiculares
  • 58. SINDROME CARCINOIDE • Cuando ocurre el síndrome carcinoide? 1) Únicamente cuando hay metástasis a hígado. 2) 10% de pacientes con tumores carcinoides • Cual es la sintomatología referida mas común de los pacientes con un tumor carcinoide que provoca un síndrome carcinoide? 1) DOLOR ABD INTERMITENTE 70% 2) DIARREA 50%: Es liquida y explosiva. 3) RUBEFACCIÓN 30%: Se caracteriza por eritema intenso o violáceo que se localiza en cara y cuello, se acompaña de sensación de calor, prurito, lagrimeo, rinorrea o edema facial, se desencadena por estrés, ejercicio, alimentos que contienen tiramina como quesos y chocolates o por la ingestión de alcohol sobre todo vino tinto. • Cual es la sustancia vasoactiva mas producida en el síndrome carcinoide? • R = La SEROTONINA sintetizada a través del triptofano, misma que es metabolizada en hígado y pulmones causando sintomatología por metastasis. • Que prueba inicial se recomienda en pacientes con sospecha de síndrome carcinoide? • R = La medición del acido 5-hidroxindolacetico (5-HIAA, metabolito de la serotonina secretado en orina) y cromogranina A (encontrada en sangre). • Cual es el manejo del síndrome carcinoide? 1) Evitar los alimentos que contienen tiramina y la ingestión de alcohol. 2) Si continua con los síntomas, los ANTAGONISTAS DE LOS RECEPTORES DE SEROTONINA (metisergida, ciproheptadina, ketanserina, ondasetron) o los ANALOGOS DE LA SOMATOSTATINA (octreotide) son los FARMACOS DE ELECCION. 3) La crisis carcinoide puede ser desencadenada por anestesia o manipulación del tumor, produciendo los mismos síntomas que el síndrome carcinoide, para evitar su aparición, antes de una intervención quirúrgica se administra octreotide de acción corta en infusión continua 12 hrs antes, durante y 48 hrs después del procedimiento.
  • 59. TRASTORNOS DE LA PUBERTAD • Cual es el primer signo físico que aparece de pubertad normal? 1) Mujeres: Botón mamario entre los 8 – 14ª 2) Hombres: Crecimiento testicular entre los 9 – 14ª • Como se define la pubertad precoz? • R = Es la aparición de caracteres sexuales secundarios ANTES DE LOS 8ª en las niñas y ANTES DE LOS 9ª en los niños. • Como realizas el diagnostico de pubertad precoz? 1) HC 2) Edad ósea, debido a que en la pubertad precoz verdadera esta se encuentra 2 desviaciones estándar por arriba para su edad y genero. 3) Laboratorios de acuerdo a sospecha clínica 4) Prueba dinámica con administración de GnRH la cual en pacientes con pubertad precoz verdadera, esperaríamos una respuesta elevada de la hormona LH. 5) El manejo deberá ir encaminado a la causa de origen • Como se define la pubertad retardada? • R = Ausencia de desarrollo de caracteres sexuales en una NINA > 13ª y en VARON > 14ª
  • 60. TRASTORNOS DE LA PUBERTAD • Cual es la causa mas frecuente de pubertad retardada? • R = RETRASO CONSTITUCIONAL, los niños que lo presentan generalmente tienen el antecedente de que alguno de sus padres también presento retraso en el inicio de la pubertad. • Como diagnosticas la pubertad retardada? 1) HC 2) Laboratorio de acuerdo a sospecha clinica • Cual es el manejo de la pubertad retardada? 1) En el retardo constitucional se vigila el momento de presentación de los caracteres secundarios, si la presión social es importante se podrán utilizar ENANTATO, CIPIONATO O PROPIONATO DE TETOSTERONA por un periodo de 3 – 6m . 2) En el caso de hipogonadismo se dará la hormona que haga falta.
  • 61. HIPERPLASIA SUPRARRENAL CONGÉNITA • A que se debe el 90 % de hiperplasia suprarrenal congénita? • R = Se debe a la imposibilidad de convertir la progesterona en 11-desoxicorticosterona mediante la 21-hidroxilasa • El déficit de la enzima 21-hidroxilasa que ocasiona fisiopatológicamente? • R = Desde la conversión de la pregenolona a aldosterona y cortisol, causando perdida de sal por una parte y por otra hiperplasia glandular por incremento del estimulo de la ACTH causando virilización.
  • 62. HIPERPLASIA SUPRARRENAL CONGÉNITA • Cual es la etiología del hiperandrogenismo suprarrenal? • R = Déficit de 21-hidroxilasa, el gen responsable se denomina CYP21B localizado en el brazo corto del cromosoma 6 • Cual es el cuadro clínico del hiperandrogenismo renal por déficit de 21-hidroxilasa? A. FORMA CLÁSICA (PERDEDORA DE SAL O VIRILIZANTE SIMPLE): Implica la existencia de hiperandrogenismo in útero que condiciona la aparición de macrogenitosomia en el varón y un grado variable de virilización en la mujer. En la forma perdedora de sal existe un DÉFICIT TANTO DE CORTISOL COMO DE ALDOSTERONA y se manifiesta en ambos sexos por perdida salina en la etapa neonatal. En la virilizante simple no esta afectada la síntesis de aldosterona por lo que se mantiene la homeostasis del sodio. B. FORMA NO CLÁSICA: Existe HIPERANDROGENISMO DE APARICIÓN POSNATAL. Los síntomas mas frecuentes durante la infancia son pubarquia prematura, piel grasa con acné, aceleración del crecimiento y maduración ósea y en las niñas aparece hipertrofia de clítoris, en la adolescencia y la edad adulta las mujeres pueden presentar trastornos menstruales, hirsutismo, acné, calvicie, ovarios poliquísticos e infertilidad, los varones afectados pueden cursar con oligoespermia, acné e infertilidad. • Como diagnosticas el déficit de la enzima 21-hidroxilasa? • R = Demostración de VALORES ELEVADOS DE 17-HIDROXIPROGESTERONA • Cual es el manejo del déficit de enzima 21-hidroxilasa? • R = GLUCOCORTICOIDES y en niños recién nacidos es FLUOROHIDROCORTISONA y en adultos es a base de PREDNISONA.
  • 63. TRASTORNOS DE LOS LIPIDOS • Cual es el hallazgo mas característico de la hiperquilomicronemia? • R = Triglicéridos mayor de 1000 • Cual es el hallazgo mas característico de la hiperbetalipoproteinemia? • R = Xantomas subcutáneos de los tendones extensores. • Cual es el hallazgo mas característico de la hiperlipoproteinemia tipo III? • R = Xantomas palmares planos.
  • 64. HEMOCROMATOSIS • Que sintomatología te sugiere hemocromatosis? 1) High ferritin, hepatomegaly, skin changes, and diabetes suggest hemochromatosis. 2) The arthritis is characterized by chondrocalcinosis, but, unlike idiopathic chondrocalcinosis, the hands are usually involved first. The arthropathy often progresses despite phlebotomy. Liver disease is usually the presenting feature. 3) Skin pigmentation is predominantly by melanin. 4) Heart failure is the most common cardiac problem.
  • 65. ENFERMEDAD DE PAGET • A 63-year-old asymptomatic woman is investigated for a HIGH ALKALINE PHOSPHATASE (ALP) level. • X-rays of the pelvis show multiple porotic and sclerotic lesions with characteristic whorls of trabeculation. Her excretion of urinary hydroxyproline is also elevated. Which of the following is the most likely diagnosis? • R = Enfermedad de Paget del Hueso • The elevated ALP AND HYDROXYPROLINE are diagnostic for Paget’s disease. The bony lesions are blastic and the sacrum and pelvis are most frequently involved, followed closely by the tibia and femur. Hypercalcemia can complícate immobilization. The etiology is unknown, but a viral agent has been postulated. Symptoms may be absent or severe (pain, deformity). In metastatic cancers of most types the lesion are lytic, and the other metabolic abnormalities do not have an elevation in hydroxyproline.
  • 66. Insulina e Hipoglucemiantes orales. Hipoglucemiantes orales. • Sulfonilureas: – 1a generación: tolbutamida (TBT), clopropramida, tolazamida, acetohexamida. – 2a generación: gliburida, glipizida, glibenclamida, glicazida. Son más potentes.
  • 67. Insulina e Hipoglucemiantes orales. Hipoglucemiantes orales. • Sulfonilureas. – Mecanismos: • Estimulan liberación de insulina, disminuyen eliminación hepática, aumentan somatostatina (disminuye glucagon). • Regulación a la baja de receptores: al susp pueden tener nuevam respuesta. • Bloq canal K sensible a ATP: despolarización: aumenta Ca.
  • 68. Insulina e Hipoglucemiantes orales. Hipoglucemiantes orales. • Sulfonilureas. – Farmacodinamia. • VO mejor 30´preprandial. • Unión PP 90-99% (alb). • T 1/2 tolazamida 4-7hrs, clorpropramida 24-48hrs. 2da generación 1.5- 5hrs (efecto disminuye glucosa x 12-24hrs). • Metab hígado. Excreción renal. Cuidado en insuf hepática y renal.
  • 69. Insulina e Hipoglucemiantes orales. Hipoglucemiantes orales. • Sulfonilureas. – Efectos colaterales. • 1a generación:40%. Menos frec 2da generación. • Hipoglucemia. • Medicamentos: desplazan de PP: sulfonamidas, clofibrato, dicumarol, salicilatos, fenilbutazona. El alcohol:aumenta su acción.. • Náusea, vómito, ictericia colestásica, agranulocitosis, A aplásica, A hemolítica, alergia. • Clorpropramida: dosulfiram 10-15%, hiponatremia 5% x aumentar ADH en túbulo colector. • TBT: aumenta 2v riesgo de muerte cardiovascular.
  • 70. Insulina e Hipoglucemiantes orales. Hipoglucemiantes orales. • Sulfonilureas. – Usos. • DMIND. • Contraindicada en DMID, embarazo, lactancia, insuf renal y hepática significativa.
  • 71. Insulina e Hipoglucemiantes orales. Hipoglucemiantes orales. • Biguanidas. – Metformin y fenformin. – Fenformin asociado a ác láctica. – Metformin T1/2 1.3-4.5 hrs. Máx 3g/d. No produce hipoglucemia, no libera insulina. – Aumenta acción perif insulina, inhibe gluconeogénesis, disminuye absorción gluc intestinal. – VO. – No en insuf renal o hepática: ác láctica, hipoxemia cr, insuf cardiaca. – Efectos colat: 20%. Diarrea, náusea, malestar abd, sabor metálico, anorexia, disminuye absorción vit B12 y folatos.
  • 72. Insulina e Hipoglucemiantes orales. Hipoglucemiantes orales. • Otros. – Ciglitazona, pioglitazona, troglitazona: thiazolidinedionas. No dan hipoglucemia. Disminuyen resistencia a insulina x aumentar transportadores glucosa. – Inhib alfa glucosidasa: acarbosa. La inhiben en el borde en cepillo: inhiben absoción CHOS. – Efectos colaterales: malaabsorción, flatulencia, distensión abd.
  • 73. SINDROME HEPATORENAL • A 44-year-old man presents with worsening abdominal distension, edema, and jaundice. He has chronic viral hepatitis B and cirrhosis. Recently he has noticed decreased urine output despite adequate fluid intake. On examination he is icteric, the blood pressure is 110/70 mm Hg, pulse 74/min, JVP is 4 cm, heart sounds are normal, and there are tense ascites and pedal edema. Which of the following is an early manifestation of hepatorenal syndrome? • R = Retención de sodio por síndrome hepatorenal • Hepatorenal syndrome frequently complicates hepatic failure. Although it can develop gradually, acute renal failure can also be precipitated by hemodynamic stresses (bleeding, diuresis). The earliest manifestations are intrarenal vasoconstriction and avid sodium retention.
  • 74. EARLY DUMPING • A 64-year-old man is placed on a low simple sugar diet. He is experiencing symptoms of crampy abdominal discomfort, nausea, diarrhea, and diaphoresis 15–30 minutes after eating. Post gastrectomía. • R = After gastrectomy, avoiding simple sugars and limiting liquids can ameliorate symptoms of dumping. Early dumping occurs within 30 minutes of eating and is characterized by vasomotor symptoms such as palpitations, tachycardia, lightheadedness, and diaphoresis. Late dumping includes similar symptoms plus dizziness, confusion, and even syncope. It occurs 1.2–3 hours after eating.
  • 75. DUNNIGAN-KOBBERLING • Que caracteriza al síndrome de Dunnigan-Kobberling tipo I? • R = Lipodistrofia parcial familiar AD, durante la pubertad los pacientes presenta atrofia de las extremidades con cara obesa. • Que caracteriza al síndrome de Dunnigan-Kobberling tipo II? • R = Atrofia de toda la grasa subcutánea o ausencia a excepción de la cara, la cual es muy obesa, pudiendo aparecer hipertrofia muscular. • Los pacientes afectados del síndrome de Dunnigan-Kobberling, a que enfermedades son propensos? • R = En especial las mujeres. Resistencia a la insulina, DM, Hipertrigliceridemia, HDL baja y aterosclerosis.
  • 76.
  • 77. GAUCHER’S DISEASE • A 26-year-old man is evaluated for back pain and fatigue. He was previously healthy. On examination, he is pale, there is lumber spine tenderness, and an enlarged liver (18 cm). He is pancytopenic, and there is a vertebral fracture on lumbar x-rays. A bone marrow biopsy reveals infiltration with lipid-laden macrophages (Gaucher cells). Which of the following is the most likely diagnosis? • Inherited Gaucher’s disease • The diagnosis is inherited Gaucher’s disease since it is an AUTOSOMAL RECESSIVE disorder. • There is no acquired form of the disease. The glucocerebrosides are derived from lipid catabolites, from the membranes of senescent leukocytes and erythrocytes. Although the juvenile form may have severe neurologic symptoms (mental retardation, spasticity, ataxia), the adult form usually has no neurologic symptoms. Like Tay-Sachs, it is a lysosomal storage disease with a predilection for Ashkenazi Jews.
  • 78. OTRAS • Cual es la forma de presentación mas común de enfermedad crónica renal relacionada a gota? • R = Disminución de la habilidad de concentración de la orina • Que tratamiento podría ayudar a una persona que padece porfiria eritropoyetica? • R = Betacaroteno aumenta la tolerancia a la luz solar. • Cuales son los síntomas de la insuficiencia adrenocortical? • R = Pérdida de peso, fatiga, debilidad nausea y vomito. • Que sintomatología ocasiona la acumulación de gangliosido y a que anormalidad metabolica esta relacionada? • Glycogen storage is not characteristic of Tay- Sachs disease. Ganglioside accumulation can now be diagnosed by decreased hexosaminidase in peripheral leukocytes. • Tay-Sachs is characterized as a lysosomal storage disease. Mental retardation, seizures, blindness, and a retinal cherry red spot are characteristic. It is most common in Ashkenazi Jews and is inherited in an autosomal recessive manner.
  • 79. OTRAS • Que entidades patológicas causan hiperpigmentación cutánea? • R = Secresion excesiva de ACTH en Addison, después de una suprarrenalectomia bilateral en la enfermedad de Cushing ¨Sindrome de Nelson¨. • Que fármacos pueden disminuir la ginecomastia? • R = Inhibidores de la aromatasa como cetrozol, anastrosol y exementano. • La galactorrea de cualquier etiología a que medicamentos responde? • R = Bromocriptina o cabergolina • Que medicamentos se administran al no encontrar testículos a la palpación? • R = HCG la cual aumenta la tetosterona
  • 80. OTRAS • Que causa y que caracteriza la enfermedad de Mc Ardle? • R = Es causada por la DISMINUCIÓN DE FOSFORILASA DE LOS MÚSCULOS. Causa debilidad, calambres musculares, concentraciones elevadas de CK. La administración de B6 disminuye síntomas. • Que causa deficiencia de ACTH a nivel supra y extrarenal? • R = Disminución de la secreción suprarrenal de cortisol, tetosterona y adrenalina. • Que caracteriza al síndrome de Kallman? • R = Causa mas común de deficiencia aislada de gonadotropina, déficit recesivo ligado al X, asociada a Hiposmia por hipoplasia de bulbos olfatorios, agenesia renal unilateral en 50%. Se trata con análogos de la GNRH como leuprolide • Cual es el estándar de oro en la disminución de la Hormona de crecimiento? • R = Hipoglucemia causada por administración de insulina • • Que complicación mas frecuente se presenta en la extirpación transfontanelar de los tumores hipofisiarios? • R = Hiponatremia
  • 81. Consejos 1. Leer diariamente y en bloques 2. Adiós Partys un tiempo 3. Has ejercicio y come bien durante el estudio 4. Toma algún curso bueno si tienes la posibilidad 5. Ten Fe.
  • 82. BIBLIOGRAFIA • EXARMED • PAPADAKIS • CTO • HARRISON • AMIR • USMLE STEPS